NURS 6521 Assignments Essay

NURS 6521 Assignments Essay

NURS 6521 week 1 Assignments Essay

Pharmacology Week 1Question 1A patient is treated with an antibiotic for an infection in his leg. After 2 days of taking the antibiotic, the patient calls the clinic and reports that he has a rash all over his body. The nurse is aware that a rash can be an adverse effect of an antibiotic and can be either a biologic, chemical, or physiologic action of the drug, which is an example ofResponse Feedback:Pharmacodynamics is the biologic, chemical, and physiologic actions of a particular drug within the body and the study of how those actions occur, including adverse effects. It is how the drug affects the body. The pharmacodynamics of a drug is responsible for its therapeutic effects and sometimes its adverse effects. Pharmacotherapeutics refers to the desired, therapeutic effect of the drug. Pharmacokinetics is the changes that occur to the drug while it is inside the body. Pharmacogenetics is the study of how genetic variables affect the pharmacodynamics of a drug in a specific patient. Question 2Which of the following statements best defines how a chemical becomes termed a drug?Response Feedback:Even though all the responses are correct, a chemical must undergo a series of tests to determine its therapeutic value and efficacy without severe toxicity or damaging properties before it is termed a drug. Test results are reported to the FDA, which may or may not give approval. Question 3A patient has been prescribed an oral drug that is known to have a high first-pass effect. Which of the following measures has the potential to increase the amount of the free drug that is available to body cells?Response Feedback:Unlike oral medications, drugs that are given intravenously do not initially pass through the liver. As a result, the first-pass effect is mitigated and more of the drug is available to cells. Frequent oral doses, low protein intake, and administration with food do not reduce the first-pass effect. Question 4The culture and sensitivity testing of a patient’s wound exudate indicates that a specific antibiotic is necessary for treatment. The United States Pharmacopeia–National Formulary indicates that the drug in question is 96% protein bound. What are the implications of this fact?Response Feedback:A drug that is 96% protein bound has only 4% of ingested molecules free and active, a fact that is likely to necessitate a high dose of the drug. This does not result in rapid absorption and/or excretion and does not indicate a need for increased protein intake. Question 5A patient has been prescribed several drugs and fluids to be given intravenously. Before the nurse starts the intravenous administration, a priority assessment of the patient will be to note theResponse Feedback:Baseline body weight and height, heart rate, and blood pressure are all important considerations during the assessment of a patient. However, if a patient has to be given drugs intravenously, it is important to inspect the skin for rashes, moles, or sores, so those areas can be avoided as an insertion or injection site. Question 6A 60-year-old African-American man lives with a number of chronic health problems. Genetic factors are likely to influence his etiology and/or treatment ofResponse Feedback:The incidence of hypertension is significantly higher among African-Americans than other ethnic groups. As well, African Americans respond to some antihypertensive drugs differently than whites. Question 7Talwin given in combination with Vistaril diminishes the adverse effects of nausea caused by the Talwin. This drug interaction affecting the pharmacodynamics of the Talwin isResponse Feedback:An antagonistic drug interaction results in a therapeutic effect that is less than the effect of either drug alone because the second drug either diminishes or cancels the effects of the first drug. An additive effect occurs when two or more “like” drugs are combined and the result is the sum of the drugs’ effects. A synergistic effect occurs when two or more “unlike” drugs are used together to produce a combined effect and the outcome is a drug effect greater than either drug’s activity alone. Potentiation is an interaction in which the effect of only one of the two drugs is increased. Question 8In response to a patient’s nausea, the nurse has mixed a dose of an antiemetic with 50 mL of sterile normal saline and will administer the dose by IV piggyback. What is the rationale for the use of IV piggyback?Response Feedback:When the patient receives continuous IV fluids and is also receiving intermittent IV drug therapy, the drug is normally given through a secondary IV tubing. When a secondary IV tubing is used to administer an IV drug, the tubing is added to the main line tubing, usually at a Y port. Adding secondary tubing is called “piggybacking” because the tubing with the drug rides on top of the primary fluid tubing. Failure to adhere to a prescribed regimen, unstable electrolyte levels, and need for continuous monitoring are not rationales for the use of an IV piggyback. Question 9The nurse is caring for a patient receiving an aminoglycoside (antibiotic) that can be nephrotoxic. Which of the following will alert the nurse that the patient may be experiencing nephrotoxicity?Response Feedback:Decreased urinary output, elevated blood urea nitrogen, increased serum creatinine, altered acid–base balance, and electrolyte imbalances can occur with nephrotoxicity. Ringing noise in the ears (tinnitus) is an indication of possible ototoxicity. Visual disturbances can suggest neurotoxicity, and yellowing of the skin (jaundice) is a sign of hepatotoxicity. Question 10A nurse is caring for a patient who has had part of her small intestine removed due to cancer. She has also now developed hypertension and has been prescribed a new medication to decrease her blood pressure. While planning the patient’s care, the nurse should consider a possible alteration in which of the following aspects of pharmacokinetics?Response Feedback:Because absorption takes place mostly in the small intestine, there could be possible alterations with this process. Distribution takes place in the blood vessels; metabolism in the liver; and elimination via the kidneys. Because these systems are not affected by her surgery, these phases of pharmacokinetics would not be altered. Question 115 ml = _______________tsp Question 12A patient who has been admitted to the hospital for a mastectomy has stated that she has experienced adverse drug effects at various times during her life. Which of the following strategies should the nurse prioritize in order to minimize the potential of adverse drug effects during the patient’s stay in the hospital?Response Feedback:In an effort to minimize the potential of adverse drug effects, it is necessary to closely monitor the patient. It would be inappropriate for the nurse to alter the route or frequency of administration or to encourage herbal remedies that also carry the potential for adverse effects. Question 13A patient with a variety of chronic health problems is being seen by her nurse practitioner, who is currently reviewing the patient’s medication regimen. Which of the patient’s medications should prompt the nurse to teach her to avoid drinking grapefruit juice?Response Feedback:Metabolism of many varied drugs such as calcium channel blockers (used to treat hypertension), statins (used to lower blood lipid levels), and antihistamines (used to prevent allergic reactions) is affected by grapefruit juice. Question 14In which of the following patients would a nurse expect to experience alterations in drug metabolism?Response Feedback:The liver is the most important site for drug metabolism. If the liver is not functioning effectively, as with cirrhosis, drugs will not be metabolized normally and toxic levels could develop. Drug dosage will have to be altered to ensure normal levels in the body. The patient with kidney stones and the patient in acute renal failure would most likely have alterations in drug excretion. If there are no complications with the cervical cancer patient, there should be no alterations in drug therapy. Question 15A patient with a recent diagnosis of acute renal failure has a long-standing seizure disorder which has been successfully controlled for several years with antiseizure medications. The nurse should recognize that the patient’s compromised renal function will likelyResponse Feedback:Impaired renal function will increase the half-life of drugs that are metabolized by the kidneys. This does not necessarily render such medications ineffective and it does not decrease the first-pass effect. IV administration will not compensate for the patient’s impaired renal function. Question 16A nurse is caring for a postsurgical patient who has small tortuous veins and had a difficult IV insertion. The patient is now receiving IV medications on a regular basis. What is the best nursing intervention to minimize the adverse effects of this drug therapy?Response Feedback:Because the patient has small tortuous veins and had a difficult IV insertion, the patient is at high risk for infiltration of the IV site. Recording baseline vital signs or blood sugar level is an important nursing action, but not specific to IV administration of any drug. The patient is not known to take anticoagulants; so unless indicated, the nurse is not required to monitor the patient’s bleeding time. Question 17An elderly postsurgical patient has developed postoperative pneumonia in the days following abdominal surgery and is being treated with a number of medications. Which of the following medications that the nurse will administer has the slowest absorption?Response Feedback:Absorption of drugs is dependent primarily on the route of administration. IV, IM, and sublingual administration results in faster absorption than drugs that are given orally. Question 18An unconscious patient has been brought to the hospital, and the physician has prescribed a life-saving drug to be administered parenterally. Which of the following methods would be the most appropriate for the nurse to use when administering the medication?Response Feedback:Intravenous infusion is the preferred method for use in emergency situations when rapid drug effects are desired. Absorption is considered to be instantaneous, as the drug is placed directly into the bloodstream. The subcutaneous and intramuscular routes could be used but would not ensure rapid drug effects. Intrathecal administration is usually done by a physician or a specially trained health care provider. Question 19The nurse’s assessment of a community-dwelling adult suggests that the client may have drug allergies that have not been previously documented. What statement by the client would confirm this?Response Feedback:True allergic reactions include formation of rash or hives, itching, redness, swelling, difficulty breathing, and anaphylactic shock. Nausea and vomiting, however, are adverse effects of drug therapy. Similarly, an unsafe drop in blood pressure and gastric bleeding from aspirin use are adverse drug effects, not allergic reactions. Question 20Tylenol 325 mg/tablet, patient needs 650 mg; how many tables should patient take? Question 21During a clinic visit, a patient complains of having frequent muscle cramps in her legs. The nurse’s assessment reveals that the patient has been taking over-the-counter laxatives for the past 7 years. The nurse informed the patient that prolonged use of laxativesResponse Feedback:Long-term intake of laxatives, antidepressants, and antibiotics has been found to deprive a person of most essential nutrients, such as vitamins. Prolonged use of laxatives is not known to turn urine acidic, cause urinary tract infections, counter the effect of other drugs, or inhibit the biotransformation of drugs. Question 22A patient who has ongoing pain issues has been prescribed meperidine (Demerol) IM. How should the nurse best administer this medication?Response Feedback:The ventrogluteal site is the preferred site for intramuscular injection. IM injections necessitate the use of a larger gauge needle than is required for subcutaneous injections. Question 23An older adult patient with a history of Alzheimer’s disease and numerous chronic health problems has been prescribed several medications during his current admission to hospital and recent declines in the patient’s cognition have impaired his ability to swallow pills. Which of the following medications may the nurse crush before administering them to this patient?Response Feedback:A tablet that is designed for immediate release into the gastric environment is normally safe to crush and administer to the patient. Enteric coated and sustained release tablets may not be crushed because doing so compromising the delayed release into the GI tract that is intended with these medications. Sublingual medications should be placed under the tongue to dissolve rather than swallowed. Question 24A nurse is discussing with a patient the efficacy of a drug that his physician has suggested, and he begin taking. Efficacy of a drug means which of the following?Response Feedback:Efficacy indicates how well a drug produces its desired effect. Different drugs have different strengths of attraction or affinity for receptor sites. A drug’s ability to stimulate its receptor is called its intrinsic activity, and the amount of a drug that must be given to produce a particular response is called the potency of a drug. Question 25A patient has been prescribed 1 mg lorazepam (Ativan) sublingual prior to the scheduled insertion of a peripherally inserted central (PIC) line. How should the nurse direct the patient when administering this medication?Response Feedback:Sublingual tablets are placed under the tongue where they dissolve and are absorbed into the bloodstream. Swallowing the pill may render it less effective, but is not unsafe. It is not recommended to chew and hold sublingual medications nor to hold them in the mouth for length of time. Question 26A nurse is caring for a 92-year-old patient who is taking multiple drugs and displaying increased cognitive impairment and memory loss. The initial action of the nurse would be toResponse Feedback:The nurse must first carefully assess and distinguish between the normal signs and symptoms of aging and the adverse effects from drug therapy, which can mimic signs of aging. The nurse may need to consult with a pharmacist to determine if symptoms stem from adverse drug reactions or drug interactions before reporting to the prescriber for treatment. Medication would not be held because of possible life-threatening conditions, and a nurse should never take a “wait and see” approach. Question 27A community health nurse is performing a home visit to an elderly client who receives twice-weekly wound care. The client has mentioned that she has been having difficulty sleeping, a problem that she has not previously experienced. Which of the following measures should the nurse suggest?Response Feedback:Whenever possible, alternatives to drug therapy should be considered as the initial treatment for problems. For an older adult with sleep difficulties, for example, it is advisable to implement non-pharmacological interventions before medications. Question 28A 90-year-old frail, elderly woman has arrived at the emergency department with a broken hip and in acute respiratory distress. Succinylcholine will be used because of the need for rapid endotracheal intubation, and then the woman will be sent to surgery. Due to the woman’s frail condition, she is at risk for skin breakdown. Which of the following nursing diagnoses would be most appropriate?Response Feedback:The nursing diagnosis that directly relates to possible skin breakdown is Impaired Physical Mobility related to drug-induced paralysis. Patients who are unable to speak, move, or breathe unassisted can quickly develop pressure sores (decubitus ulcers). It is the responsibility of the nurse to help keep the patient positioned correctly to avoid skin breakdown. The nurse will plan interventions to prevent skin breakdown based on how long the patient will be immobilized. Impaired Spontaneous Ventilation is related to respiratory paralysis. Fear is related to paralysis and helplessness, and Disturbed Sensory Perception is related to CNS depression secondary to drugs used during anesthesia. Question 29An 80-year-old man has been prescribed oxycodone for severe, noncancer, chronic pain. He tells the nurse that he has difficulty swallowing and asks if he can crush the tablet before swallowing. The nurse will advise the patient thatResponse Feedback:The nurse should caution the patient against crushing the tablet before ingesting it. Crushing allows an extremely high dose of the drug to be available all at once, instead of being released slowly over time. Severe adverse effects are possible when it is used in this manner. The other options are false statements. Question 30A 70-year-old man who enjoys good health began taking low-dose aspirin several months ago based on recommendations that he read in a magazine article. During the man’s most recent visit to his care provider, routine blood work was ordered and the results indicated an unprecedented rise in the man’s serum creatinine and blood urea nitrogen (BUN) levels. How should a nurse best interpret these findings?Response Feedback:Damage to the kidneys is called nephrotoxicity. Decreased urinary output, elevated blood urea nitrogen, increased serum creatinine, altered acid-base balance, and electrolyte imbalances can all occur with kidney damage. Question 31Medication reconciliation of an 82-year-old man who has recently moved to a long-term care facility reveals that the man takes 1 to 2 mg of lorazepam bid prn. The nurse should recognize what consequence of this aspect of the resident’s drug regimen?Response Feedback:In a systematic review of medications as risk factors for fall, it was found that one of the main group of drugs associated with this risk were benzodiazepines. Benzodiazepines are not associated with cold intolerance, anorexia, or aggression. Question 32A 72-year-old man is taking Adderall XR for the treatment of narcolepsy. He is currently having problems with not being able to swallow large tablets or capsules. The man also wears dentures, which makes it even more difficult for him to swallow medication. He is in the clinic to talk to the nurse about his problem. The nurse will instruct him toResponse Feedback:The benefits of Adderall XR are its once-daily dosing, its longer duration of action, and its potential for sprinkle administration. For patients with difficulty swallowing, Adderall XR’s capsule may be opened and the beads sprinkled in applesauce. It is not advisable to suggest the use of an alternative drug. Patients should be told not to crush the beads after opening the capsule because this would alter drug absorption. Ingesting the capsule with 8 ounces of water will not solve the patient’s difficulty with swallowing. Question 33A nurse is administering drugs to a 70-year-old patient who has a reduced plasma albumin level. When assessing the patient for therapeutic outcomes of drug therapy, the nurse will also be careful to observe forResponse Feedback:Decreased levels of plasma albumin can lead to fewer binding sites for protein-binding drug molecules. Decreased binding sites results in higher concentrations of unbound forms of a drug, which increases the risk of adverse effects and toxicity and can increase therapeutic effects. Plasma albumin does not alter absorption of a drug but is responsible for binding, transporting, and distributing drugs throughout the body. Even though higher concentrations of a free drug increase the amount of the drug available for metabolism, normal age-related decreases in liver function offset any increase in the pharmacokinetic process. Question 34A nurse is caring for a 78-year-old patient with renal insufficiency and chronic heart failure who is receiving rapid infusions of high-dose furosemide. It will be a priority for the nurse to monitor forResponse Feedback:Ototoxicity can occur with rapid intravenous administration, especially in patients with poor renal function and in those receiving high doses of furosemide. Although usually transient, ototoxicity may result in permanent damage. Rapid infusions of high-dose furosemide would not place the patient at risk for hepatic encephalopathy. Excessive diuresis from furosemide can result in dehydration and vascular thrombosis, but they would not be the priority in this case. Question 35A nurse is caring for a 73-year-old man who is receiving drug therapy. He is beginning to exhibit signs of decline in his renal system, yet his current serum creatinine level is normal. The nurse will base the patient’s plan of care on the understanding that there isResponse Feedback:The patient’s creatinine level is within the normal range for his age. Less creatinine overall exists in the older adult because creatinine production declines as muscle mass decreases. The normal creatinine level can be misleading and should not be interpreted as an indication of normal renal function or effectiveness of the drug. Question 36Mr. Lacuna is an 83-year-old resident of a long-term care facility who has a diagnosis of moderate Alzheimer disease. Mr. Lacuna’s physician recently prescribed oral rivastigmine, but he was unable to tolerate the drug due to its gastrointestinal effects. As a result, he has been ordered the transdermal patch form of the medication. When administering this form of rivastigmine, the nurse shouldResponse Feedback:In patients with moderate dementia, it may be necessary to place the patch on the back where it cannot be removed in settings where a nurse will be reapplying the patch. A patient with moderate dementia would not normally self-manage medications. The dosage of rivastigmine is not increased during a short-term exacerbation of symptoms. Transdermal patches are never applied to mucous membranes. Question 37Mrs. Houston is a 78-year-old woman who resides in an assisted living facility. Her doctor prescribed digoxin at her last visit to the clinic and she has approached the nurse who makes regular visits to the assisted-living facility about this new drug. What teaching point should the nurse emphasize to Mrs. Houston?Response Feedback:Patients taking digoxin need to know the importance of having all laboratory work (serum drug levels; electrolytes) performed on time. Digoxin is not taken if the patient is bradycardic, and it does not provide an immediate or short-term increase in exercise tolerance. Blood pressure monitoring is not normally required for patients who are taking digoxin. Question 38A 79-year-old woman has been admitted to the hospital with a gastrointestinal bleed. She is currently receiving a transfusion of packed red blood cells as well as an infusion of pantoprazole (Protonix), a proton-pump inhibitor, to help stop her bleeding. What care setting is most appropriate for this patient?Response Feedback:Blood transfusions and continuous infusions of intravenous medications require the vigilant assessment and monitoring that can be provided in an acute care setting; this is not normally possible in an ARU or transitional care setting and would be impossible in an outpatient environment. Question 39A 77-year-old man with a long history of absence seizures has been treated with ethosuximide for many years. The man is now in the process of moving to a long-term care facility and a nurse is creating a plan of care. The nurse understands the potential adverse effects of this drug and would consequently prioritize which of the following nursing diagnoses?Response Feedback:Common adverse effects of ethosuximide are drowsiness, dizziness, and lethargy. Respiration, urinary function, and skin integrity are not normally affected by the use of ethosuximide. Question 40A 66-year-old woman has a complex medical history that includes poorly-controlled type 1 diabetes, renal failure as a result of diabetic nephropathy and chronic heart failure (CHF). Her care provider has recently added spironolactone (Aldactone) to the woman’s medication regimen. The nurse should consequently assess for signs and symptoms ofResponse Feedback:Spironolactone (Aldactone) and eplerenone (Inspra) are potassium-sparing diuretics, so their use increases serum potassium levels. Spironolactone is not known to contribute to dysrhythmias, leukocytosis, or thrombophlebitis

ORDER  HERE NOW

NURS 6521 week 2 Assignments Essay

NURS 6521N Week 2 Quiz (2018): Advanced Pharmacology: Walden University

Question 1
A patient with seasonal allergies is exasperated by her recent nasal congestion and has expressed her desire to treat it by using pseudoephedrine. The nurse should inform the patient that

A.
drugs containing pseudoephedrine are available without a prescription but are kept off pharmacy shelves.

B.
she must be prepared to show identification when buying a drug that contains pseudoephedrine.

C.
pseudoephedrine is being phased out of the marketplace in order to combat illegal use.

D.
under the Combat Methamphetamine Abuse Act, pseudoephedrine now requires a prescription.

1 points
Question 2
A nurse is caring for a 46-year-old patient of Chinese origin who has bipolar disorder. The physician has prescribed lithium carbonate (Eskalith) to treat the disorder. The nurse is aware that the lithium dose will likely be given in a

A.
weight-based protocol dose.

B.
higher-than-normal dose.

C.
lower-than-normal dose.

D.
child’s dose.

1 points
Question 3
A nurse has just completed a medication history on a newly admitted patient. In order to complete medication reconciliation for this patient the nurse will

A.
Provide instructions pertaining to each medication the patient is currently taking and then add the medications ordered during the hospitalization.

B.
explain to the patient the pharmacokinetics of each drug he will be taking in the hospital.

C.
compare the medications ordered by the health care provider with the list of medications obtained from the patient and communicate discrepancies to the health care provider.

D.
determine the best pharmacy for the patient to buy his medications.

1 points
Question 4
Which of the following patients should be advised by the nurse to avoid over-the-counter cold and allergy preparations that contain phenylephrine?

A.
A 52-year-old male with adult-onset diabetes

B.
A 62-year-old male with gout

C.
A 47-year-old female with hypertension

D.
A 17-year-old female with symptoms of an upper respiratory infection

1 points
Question 5
A patient has taken an overdose of a vitamin/mineral supplement containing magnesium. The nurse will be sure to assess

A.
body temperature.

B.
blood pressure.

C.
fluid intake.

D.
skin changes.

1 points
Question 6
A nurse practitioner understands when prescribing a medication that there are certain questions to address. Check all that apply.

A.
Is there a need for the drug in treating the presenting problem?

B.
Is this the best drug for the presenting problem?

C.
Can the patient take the prescribed drug?

D.
None of the above.

1 points
Question 7
A patient has been prescribed a drug that can be self-administered at home. Which of the following would be the most important information for the nurse to relate to the patient concerning self-administration of a drug?

A.
The pharmacokinetics of the drug

B.
The therapeutic effects and possible adverse effects

C.
The cost of the drug

D.
Where to purchase the medication

1 points
Question 8
What critical piece of information is missing from the following medication order: Amoxicillin 250 mg every 8 hours?

A.
Dose

B.
Route

C.
Frequency

D.
Trade name of the drug

1 points
Question 9
A patient has been prescribed a Scheduled 5 drug, an example of this drug is

A.
Antianxiety

B.
Narcotic Analgesics

C.
Barbituates

D.
Antitussives

1 points
Question 10
A nurse is providing a patient with a list of drugs as a part of the patient’s plan of care. Which of the following drug nomenclatures should the nurse use to list the drugs?

A.
The drugs’ biologic names

B.
The drugs’ chemical names

C.
The drugs’ trade names

D.
The drugs’ generic names

1 points
Question 11
These drugs have a potential for abuse, but the potential is lower than for drugs on Schedule 2. These drugs contain a combination of controlled and noncontrolled substances. Use of these drugs can cause a moderate to low physiologic dependence and a higher psychological dependence. A verbal order can be given to the pharmacy and the prescription can be refilled up to five times within 6 months. Examples include certain narcotics (codeine) and nonbarbiturate sedatives. This is scheduled drug

A.
Schedule 1

B.
Schedule 2

C.
Schedule 3

D.
Schedule 4

1 points
Question 12
A nurse is a member of a research team that is exploring unique differences in responses to drugs that each individual possesses, based on genetic make-up. This area of study is called

A.
pharmacotherapeutics.

B.
pharmacoeconomics.

C.
pharmacodynamics.

D.
pharmacogenomics.

1 points
Question 13
A nurse working for a drug company is involved in phase III drug evaluation studies. Which of the following might the nurse be responsible for during this stage of drug development?

A.
Monitoring drug effects in patients who are selected to participate in a study, who have the disease that the drug is meant to treat

B.
Administering investigational drugs to patients

C.
Informing healthy, young volunteer participants of possible risks that could occur from taking an experimental drug

D.
Working with animals who are given experimental drugs

1 points
Question 14
A patient will begin three new medications as part of her treatment plan. The nurse practitioner understands that proper disposal of medications is key when the nurse practitioner states

A.
Many medications can be potentially harmful if taken by someone other than the person the medication was prescribed.

B.
Consult with several pharmacists to determine the cost of each drug.

C.
educate the patient to discard syringes in recycle bin

D.
consult with the physician concerning choice of medications for the patient.

1 points
Question 15
The nurse practitioner orders Amoxicillin 500 mg tid? What is the total amount of medication patient will take per day?

A.
500 mg

B.
1000 mg

C.
1500 mg

D.
150 mg

1 points
Question 16
Drugs have a valid medical use but a high potential for abuse, both psychological and physiologic. In an emergency, a Schedule 2 drug may be prescribed by telephone if a written prescription cannot be provided at the time. However, a written prescription must be provided within 72 hours with the words authorization for emergency dispensing written on the prescription. These prescriptions cannot be refilled. A new prescription must be written each time. Examples include certain amphetamines and barbiturates. This is scheduled drug

A.
Schedule 1

B.
Schedule 2

C.
Schedule 3

D.
Schedule 4

1 points
Question 17
A nurse receives an order to administer a critically ill patient two drugs immediately (stat). The nurse begins the process by

A.
consulting a drug guide for compatibility.

B.
identifying the patient by checking the armband and asking the patient to state his name.

C.
questioning the patient concerning allergies.

D.
washing his or her hands before handling the medications.

1 points
Question 18
Which of the following serves to protect the public by ensuring the purity of a drug and its contents?

A.
Federal legislation

B.
United States Adopted Names Council

C.
Nursing drug guides

D.
American Pharmaceutical Association

1 points
Question 19
A Native American man who lives a traditional lifestyle is scheduled to have heart surgery. The tribal chief has requested that the tribe’s medicine man perform a ritual before the patient goes to surgery. The nurse’s response to this request should be

A.
“I don’t think that will help, but you have the right to perform the ritual.”

B.
“I’m sorry, but that will not be possible.”

C.
“I will need to inform the doctor and see if he will allow this.”

D.
“As long as the ritual does not interfere with the patient’s preoperative care, it will be okay.”

1 points
Question 20
A nurse is admitting a Mexican woman to the hospital who cannot speak or understand English. The patient is alone, and there is no interpreter available. When trying to communicate with the patient the nurse will

A.
use nonverbal language.

B.
exaggerate her mouth movements.

C.
speak loudly.

D.
seak fast.

1 points
Question 21
A physician has ordered subcutaneous injections of morphine, a narcotic, every 4 hours as needed for pain for a motor vehicle accident victim. The nurse is aware that there is a high abuse potential for this drug and that it is categorized as a

A.
C-IV drug.

B.
C-II drug.

C.
C-III drug.

D.
C-I drug.

1 points
Question 22
A nurse who provides care in a long-term care facility is documenting a new resident’s medication regimen on the resident’s intake admission. The nurse is documenting the generic, rather than proprietary, names of the resident’s current drugs because

A.
documentation of trade names for drugs requires permission from the manufacturer.

B.
there is a potential for confusion and miscommunication if proprietary names are used.

C.
drugs with generic names are prescribed and used differently than proprietary drugs.

D.
proprietary names of drugs can vary from state to state.

1 points
Question 23
A patient comes to primary clinic for strep throat. A throat swab culture is sent to lab. What information is required for the nurse practitioner to disclose on lab transmittal?

A.
National Provider Identification (NPI)

B.
Centers for Medicare & Medicaid Services (CMS)

C.
Health Insurance Portability and Accountability Act of 1996 (HIPPA)

D.
None of the above

1 points
Question 24
A 46-year-old white American has been prescribed a drug that binds to acid glycoproteins. The nurse understands that white Americans usually receive

A.
the recommended normal dose of drugs that bind to acid glycoproteins.

B.
a higher-than-normal dose of drugs that bind to acid glycoproteins.

C.
a lower-than-normal dose of drugs that bind to acid glycoproteins.

D.
one half the recommended dose of drugs that bind to acid glycoproteins.

1 points
Question 25
Drugs have a high potential for abuse. There is no routine therapeutic use for these drugs and they are not available for regular use. They may be obtained for “investigational use only” by applying to the U.S. Drug Enforcement Agency. Examples include heroin and LSD. Which scheduled drug is this?

A.
Schedule 1

B.
Schedule 2

C.
Schedule 3

D.
Schedule 4

NURS 6521N Week 2 Quiz (2018): Advanced Pharmacology: Walden University

Question 1
A patient with seasonal allergies is exasperated by her recent nasal congestion and has expressed her desire to treat it by using pseudoephedrine. The nurse should inform the patient that

A.
drugs containing pseudoephedrine are available without a prescription but are kept off pharmacy shelves.

B.
she must be prepared to show identification when buying a drug that contains pseudoephedrine.

C.
pseudoephedrine is being phased out of the marketplace in order to combat illegal use.

D.
under the Combat Methamphetamine Abuse Act, pseudoephedrine now requires a prescription.

1 points
Question 2
A nurse is caring for a 46-year-old patient of Chinese origin who has bipolar disorder. The physician has prescribed lithium carbonate (Eskalith) to treat the disorder. The nurse is aware that the lithium dose will likely be given in a

A.
weight-based protocol dose.

B.
higher-than-normal dose.

C.
lower-than-normal dose.

D.
child’s dose.

1 points
Question 3
A nurse has just completed a medication history on a newly admitted patient. In order to complete medication reconciliation for this patient the nurse will

A.
Provide instructions pertaining to each medication the patient is currently taking and then add the medications ordered during the hospitalization.

B.
explain to the patient the pharmacokinetics of each drug he will be taking in the hospital.

C.
compare the medications ordered by the health care provider with the list of medications obtained from the patient and communicate discrepancies to the health care provider.

D.
determine the best pharmacy for the patient to buy his medications.

1 points
Question 4
Which of the following patients should be advised by the nurse to avoid over-the-counter cold and allergy preparations that contain phenylephrine?

A.
A 52-year-old male with adult-onset diabetes

B.
A 62-year-old male with gout

C.
A 47-year-old female with hypertension

D.
A 17-year-old female with symptoms of an upper respiratory infection

1 points
Question 5
A patient has taken an overdose of a vitamin/mineral supplement containing magnesium. The nurse will be sure to assess

A.
body temperature.

B.
blood pressure.

C.
fluid intake.

D.
skin changes.

1 points
Question 6
A nurse practitioner understands when prescribing a medication that there are certain questions to address. Check all that apply.

A.
Is there a need for the drug in treating the presenting problem?

B.
Is this the best drug for the presenting problem?

C.
Can the patient take the prescribed drug?

D.
None of the above.

1 points
Question 7
A patient has been prescribed a drug that can be self-administered at home. Which of the following would be the most important information for the nurse to relate to the patient concerning self-administration of a drug?

A.
The pharmacokinetics of the drug

B.
The therapeutic effects and possible adverse effects

C.
The cost of the drug

D.
Where to purchase the medication

1 points
Question 8
What critical piece of information is missing from the following medication order: Amoxicillin 250 mg every 8 hours?

A.
Dose

B.
Route

C.
Frequency

D.
Trade name of the drug

1 points
Question 9
A patient has been prescribed a Scheduled 5 drug, an example of this drug is

A.
Antianxiety

B.
Narcotic Analgesics

C.
Barbituates

D.
Antitussives

1 points
Question 10
A nurse is providing a patient with a list of drugs as a part of the patient’s plan of care. Which of the following drug nomenclatures should the nurse use to list the drugs?

A.
The drugs’ biologic names

B.
The drugs’ chemical names

C.
The drugs’ trade names

D.
The drugs’ generic names

1 points
Question 11
These drugs have a potential for abuse, but the potential is lower than for drugs on Schedule 2. These drugs contain a combination of controlled and noncontrolled substances. Use of these drugs can cause a moderate to low physiologic dependence and a higher psychological dependence. A verbal order can be given to the pharmacy and the prescription can be refilled up to five times within 6 months. Examples include certain narcotics (codeine) and nonbarbiturate sedatives. This is scheduled drug

A.
Schedule 1

B.
Schedule 2

C.
Schedule 3

D.
Schedule 4

1 points
Question 12
A nurse is a member of a research team that is exploring unique differences in responses to drugs that each individual possesses, based on genetic make-up. This area of study is called

A.
pharmacotherapeutics.

B.
pharmacoeconomics.

C.
pharmacodynamics.

D.
pharmacogenomics.

1 points
Question 13
A nurse working for a drug company is involved in phase III drug evaluation studies. Which of the following might the nurse be responsible for during this stage of drug development?

A.
Monitoring drug effects in patients who are selected to participate in a study, who have the disease that the drug is meant to treat

B.
Administering investigational drugs to patients

C.
Informing healthy, young volunteer participants of possible risks that could occur from taking an experimental drug

D.
Working with animals who are given experimental drugs

1 points
Question 14
A patient will begin three new medications as part of her treatment plan. The nurse practitioner understands that proper disposal of medications is key when the nurse practitioner states

A.
Many medications can be potentially harmful if taken by someone other than the person the medication was prescribed.

B.
Consult with several pharmacists to determine the cost of each drug.

C.
educate the patient to discard syringes in recycle bin

D.
consult with the physician concerning choice of medications for the patient.

1 points
Question 15
The nurse practitioner orders Amoxicillin 500 mg tid? What is the total amount of medication patient will take per day?

A.
500 mg

B.
1000 mg

C.
1500 mg

D.
150 mg

1 points
Question 16
Drugs have a valid medical use but a high potential for abuse, both psychological and physiologic. In an emergency, a Schedule 2 drug may be prescribed by telephone if a written prescription cannot be provided at the time. However, a written prescription must be provided within 72 hours with the words authorization for emergency dispensing written on the prescription. These prescriptions cannot be refilled. A new prescription must be written each time. Examples include certain amphetamines and barbiturates. This is scheduled drug

A.
Schedule 1

B.
Schedule 2

C.
Schedule 3

D.
Schedule 4

1 points
Question 17
A nurse receives an order to administer a critically ill patient two drugs immediately (stat). The nurse begins the process by

A.
consulting a drug guide for compatibility.

B.
identifying the patient by checking the armband and asking the patient to state his name.

C.
questioning the patient concerning allergies.

D.
washing his or her hands before handling the medications.

1 points
Question 18
Which of the following serves to protect the public by ensuring the purity of a drug and its contents?

A.
Federal legislation

B.
United States Adopted Names Council

C.
Nursing drug guides

D.
American Pharmaceutical Association

1 points
Question 19
A Native American man who lives a traditional lifestyle is scheduled to have heart surgery. The tribal chief has requested that the tribe’s medicine man perform a ritual before the patient goes to surgery. The nurse’s response to this request should be

A.
“I don’t think that will help, but you have the right to perform the ritual.”

B.
“I’m sorry, but that will not be possible.”

C.
“I will need to inform the doctor and see if he will allow this.”

D.
“As long as the ritual does not interfere with the patient’s preoperative care, it will be okay.”

1 points
Question 20
A nurse is admitting a Mexican woman to the hospital who cannot speak or understand English. The patient is alone, and there is no interpreter available. When trying to communicate with the patient the nurse will

A.
use nonverbal language.

B.
exaggerate her mouth movements.

C.
speak loudly.

D.
seak fast.

1 points
Question 21
A physician has ordered subcutaneous injections of morphine, a narcotic, every 4 hours as needed for pain for a motor vehicle accident victim. The nurse is aware that there is a high abuse potential for this drug and that it is categorized as a

A.
C-IV drug.

B.
C-II drug.

C.
C-III drug.

D.
C-I drug.

1 points
Question 22
A nurse who provides care in a long-term care facility is documenting a new resident’s medication regimen on the resident’s intake admission. The nurse is documenting the generic, rather than proprietary, names of the resident’s current drugs because

A.
documentation of trade names for drugs requires permission from the manufacturer.

B.
there is a potential for confusion and miscommunication if proprietary names are used.

C.
drugs with generic names are prescribed and used differently than proprietary drugs.

D.
proprietary names of drugs can vary from state to state.

1 points
Question 23
A patient comes to primary clinic for strep throat. A throat swab culture is sent to lab. What information is required for the nurse practitioner to disclose on lab transmittal?

A.
National Provider Identification (NPI)

B.
Centers for Medicare & Medicaid Services (CMS)

C.
Health Insurance Portability and Accountability Act of 1996 (HIPPA)

D.
None of the above

1 points
Question 24
A 46-year-old white American has been prescribed a drug that binds to acid glycoproteins. The nurse understands that white Americans usually receive

A.
the recommended normal dose of drugs that bind to acid glycoproteins.

B.
a higher-than-normal dose of drugs that bind to acid glycoproteins.

C.
a lower-than-normal dose of drugs that bind to acid glycoproteins.

D.
one half the recommended dose of drugs that bind to acid glycoproteins.

1 points
Question 25
Drugs have a high potential for abuse. There is no routine therapeutic use for these drugs and they are not available for regular use. They may be obtained for “investigational use only” by applying to the U.S. Drug Enforcement Agency. Examples include heroin and LSD. Which scheduled drug is this?

A.
Schedule 1

B.
Schedule 2

C.
Schedule 3

D.
Schedule 4

NURS 6521N Week 2 Quiz (2018): Advanced Pharmacology: Walden University

Question 1
A patient with seasonal allergies is exasperated by her recent nasal congestion and has expressed her desire to treat it by using pseudoephedrine. The nurse should inform the patient that

A.
drugs containing pseudoephedrine are available without a prescription but are kept off pharmacy shelves.

B.
she must be prepared to show identification when buying a drug that contains pseudoephedrine.

C.
pseudoephedrine is being phased out of the marketplace in order to combat illegal use.

D.
under the Combat Methamphetamine Abuse Act, pseudoephedrine now requires a prescription.

1 points
Question 2
A nurse is caring for a 46-year-old patient of Chinese origin who has bipolar disorder. The physician has prescribed lithium carbonate (Eskalith) to treat the disorder. The nurse is aware that the lithium dose will likely be given in a

A.
weight-based protocol dose.

B.
higher-than-normal dose.

C.
lower-than-normal dose.

D.
child’s dose.

1 points
Question 3
A nurse has just completed a medication history on a newly admitted patient. In order to complete medication reconciliation for this patient the nurse will

A.
Provide instructions pertaining to each medication the patient is currently taking and then add the medications ordered during the hospitalization.

B.
explain to the patient the pharmacokinetics of each drug he will be taking in the hospital.

C.
compare the medications ordered by the health care provider with the list of medications obtained from the patient and communicate discrepancies to the health care provider.

D.
determine the best pharmacy for the patient to buy his medications.

1 points
Question 4
Which of the following patients should be advised by the nurse to avoid over-the-counter cold and allergy preparations that contain phenylephrine?

A.
A 52-year-old male with adult-onset diabetes

B.
A 62-year-old male with gout

C.
A 47-year-old female with hypertension

D.
A 17-year-old female with symptoms of an upper respiratory infection

1 points
Question 5
A patient has taken an overdose of a vitamin/mineral supplement containing magnesium. The nurse will be sure to assess

A.
body temperature.

B.
blood pressure.

C.
fluid intake.

D.
skin changes.

1 points
Question 6
A nurse practitioner understands when prescribing a medication that there are certain questions to address. Check all that apply.

A.
Is there a need for the drug in treating the presenting problem?

B.
Is this the best drug for the presenting problem?

C.
Can the patient take the prescribed drug?

D.
None of the above.

1 points
Question 7
A patient has been prescribed a drug that can be self-administered at home. Which of the following would be the most important information for the nurse to relate to the patient concerning self-administration of a drug?

A.
The pharmacokinetics of the drug

B.
The therapeutic effects and possible adverse effects

C.
The cost of the drug

D.
Where to purchase the medication

1 points
Question 8
What critical piece of information is missing from the following medication order: Amoxicillin 250 mg every 8 hours?

A.
Dose

B.
Route

C.
Frequency

D.
Trade name of the drug

1 points
Question 9
A patient has been prescribed a Scheduled 5 drug, an example of this drug is

A.
Antianxiety

B.
Narcotic Analgesics

C.
Barbituates

D.
Antitussives

1 points
Question 10
A nurse is providing a patient with a list of drugs as a part of the patient’s plan of care. Which of the following drug nomenclatures should the nurse use to list the drugs?

A.
The drugs’ biologic names

B.
The drugs’ chemical names

C.
The drugs’ trade names

D.
The drugs’ generic names

1 points
Question 11
These drugs have a potential for abuse, but the potential is lower than for drugs on Schedule 2. These drugs contain a combination of controlled and noncontrolled substances. Use of these drugs can cause a moderate to low physiologic dependence and a higher psychological dependence. A verbal order can be given to the pharmacy and the prescription can be refilled up to five times within 6 months. Examples include certain narcotics (codeine) and nonbarbiturate sedatives. This is scheduled drug

A.
Schedule 1

B.
Schedule 2

C.
Schedule 3

D.
Schedule 4

1 points
Question 12
A nurse is a member of a research team that is exploring unique differences in responses to drugs that each individual possesses, based on genetic make-up. This area of study is called

A.
pharmacotherapeutics.

B.
pharmacoeconomics.

C.
pharmacodynamics.

D.
pharmacogenomics.

1 points
Question 13
A nurse working for a drug company is involved in phase III drug evaluation studies. Which of the following might the nurse be responsible for during this stage of drug development?

A.
Monitoring drug effects in patients who are selected to participate in a study, who have the disease that the drug is meant to treat

B.
Administering investigational drugs to patients

C.
Informing healthy, young volunteer participants of possible risks that could occur from taking an experimental drug

D.
Working with animals who are given experimental drugs

1 points
Question 14
A patient will begin three new medications as part of her treatment plan. The nurse practitioner understands that proper disposal of medications is key when the nurse practitioner states

A.
Many medications can be potentially harmful if taken by someone other than the person the medication was prescribed.

B.
Consult with several pharmacists to determine the cost of each drug.

C.
educate the patient to discard syringes in recycle bin

D.
consult with the physician concerning choice of medications for the patient.

1 points
Question 15
The nurse practitioner orders Amoxicillin 500 mg tid? What is the total amount of medication patient will take per day?

A.
500 mg

B.
1000 mg

C.
1500 mg

D.
150 mg

1 points
Question 16
Drugs have a valid medical use but a high potential for abuse, both psychological and physiologic. In an emergency, a Schedule 2 drug may be prescribed by telephone if a written prescription cannot be provided at the time. However, a written prescription must be provided within 72 hours with the words authorization for emergency dispensing written on the prescription. These prescriptions cannot be refilled. A new prescription must be written each time. Examples include certain amphetamines and barbiturates. This is scheduled drug

A.
Schedule 1

B.
Schedule 2

C.
Schedule 3

D.
Schedule 4

1 points
Question 17
A nurse receives an order to administer a critically ill patient two drugs immediately (stat). The nurse begins the process by

A.
consulting a drug guide for compatibility.

B.
identifying the patient by checking the armband and asking the patient to state his name.

C.
questioning the patient concerning allergies.

D.
washing his or her hands before handling the medications.

1 points
Question 18
Which of the following serves to protect the public by ensuring the purity of a drug and its contents?

A.
Federal legislation

B.
United States Adopted Names Council

C.
Nursing drug guides

D.
American Pharmaceutical Association

1 points
Question 19
A Native American man who lives a traditional lifestyle is scheduled to have heart surgery. The tribal chief has requested that the tribe’s medicine man perform a ritual before the patient goes to surgery. The nurse’s response to this request should be

A.
“I don’t think that will help, but you have the right to perform the ritual.”

B.
“I’m sorry, but that will not be possible.”

C.
“I will need to inform the doctor and see if he will allow this.”

D.
“As long as the ritual does not interfere with the patient’s preoperative care, it will be okay.”

1 points
Question 20
A nurse is admitting a Mexican woman to the hospital who cannot speak or understand English. The patient is alone, and there is no interpreter available. When trying to communicate with the patient the nurse will

A.
use nonverbal language.

B.
exaggerate her mouth movements.

C.
speak loudly.

D.
seak fast.

1 points
Question 21
A physician has ordered subcutaneous injections of morphine, a narcotic, every 4 hours as needed for pain for a motor vehicle accident victim. The nurse is aware that there is a high abuse potential for this drug and that it is categorized as a

A.
C-IV drug.

B.
C-II drug.

C.
C-III drug.

D.
C-I drug.

1 points
Question 22
A nurse who provides care in a long-term care facility is documenting a new resident’s medication regimen on the resident’s intake admission. The nurse is documenting the generic, rather than proprietary, names of the resident’s current drugs because

A.
documentation of trade names for drugs requires permission from the manufacturer.

B.
there is a potential for confusion and miscommunication if proprietary names are used.

C.
drugs with generic names are prescribed and used differently than proprietary drugs.

D.
proprietary names of drugs can vary from state to state.

1 points
Question 23
A patient comes to primary clinic for strep throat. A throat swab culture is sent to lab. What information is required for the nurse practitioner to disclose on lab transmittal?

A.
National Provider Identification (NPI)

B.
Centers for Medicare & Medicaid Services (CMS)

C.
Health Insurance Portability and Accountability Act of 1996 (HIPPA)

D.
None of the above

1 points
Question 24
A 46-year-old white American has been prescribed a drug that binds to acid glycoproteins. The nurse understands that white Americans usually receive

A.
the recommended normal dose of drugs that bind to acid glycoproteins.

B.
a higher-than-normal dose of drugs that bind to acid glycoproteins.

C.
a lower-than-normal dose of drugs that bind to acid glycoproteins.

D.
one half the recommended dose of drugs that bind to acid glycoproteins.

1 points
Question 25
Drugs have a high potential for abuse. There is no routine therapeutic use for these drugs and they are not available for regular use. They may be obtained for “investigational use only” by applying to the U.S. Drug Enforcement Agency. Examples include heroin and LSD. Which scheduled drug is this?

A.
Schedule 1

B.
Schedule 2

C.
Schedule 3

D.
Schedule 4

NURS 6521N Week 2 Quiz (2018): Advanced Pharmacology: Walden University

Question 1
A patient with seasonal allergies is exasperated by her recent nasal congestion and has expressed her desire to treat it by using pseudoephedrine. The nurse should inform the patient that

A.
drugs containing pseudoephedrine are available without a prescription but are kept off pharmacy shelves.

B.
she must be prepared to show identification when buying a drug that contains pseudoephedrine.

C.
pseudoephedrine is being phased out of the marketplace in order to combat illegal use.

D.
under the Combat Methamphetamine Abuse Act, pseudoephedrine now requires a prescription.

1 points
Question 2
A nurse is caring for a 46-year-old patient of Chinese origin who has bipolar disorder. The physician has prescribed lithium carbonate (Eskalith) to treat the disorder. The nurse is aware that the lithium dose will likely be given in a

A.
weight-based protocol dose.

B.
higher-than-normal dose.

C.
lower-than-normal dose.

D.
child’s dose.

1 points
Question 3
A nurse has just completed a medication history on a newly admitted patient. In order to complete medication reconciliation for this patient the nurse will

A.
Provide instructions pertaining to each medication the patient is currently taking and then add the medications ordered during the hospitalization.

B.
explain to the patient the pharmacokinetics of each drug he will be taking in the hospital.

C.
compare the medications ordered by the health care provider with the list of medications obtained from the patient and communicate discrepancies to the health care provider.

D.
determine the best pharmacy for the patient to buy his medications.

1 points
Question 4
Which of the following patients should be advised by the nurse to avoid over-the-counter cold and allergy preparations that contain phenylephrine?

A.
A 52-year-old male with adult-onset diabetes

B.
A 62-year-old male with gout

C.
A 47-year-old female with hypertension

D.
A 17-year-old female with symptoms of an upper respiratory infection

1 points
Question 5
A patient has taken an overdose of a vitamin/mineral supplement containing magnesium. The nurse will be sure to assess

A.
body temperature.

B.
blood pressure.

C.
fluid intake.

D.
skin changes.

1 points
Question 6
A nurse practitioner understands when prescribing a medication that there are certain questions to address. Check all that apply.

A.
Is there a need for the drug in treating the presenting problem?

B.
Is this the best drug for the presenting problem?

C.
Can the patient take the prescribed drug?

D.
None of the above.

1 points
Question 7
A patient has been prescribed a drug that can be self-administered at home. Which of the following would be the most important information for the nurse to relate to the patient concerning self-administration of a drug?

A.
The pharmacokinetics of the drug

B.
The therapeutic effects and possible adverse effects

C.
The cost of the drug

D.
Where to purchase the medication

1 points
Question 8
What critical piece of information is missing from the following medication order: Amoxicillin 250 mg every 8 hours?

A.
Dose

B.
Route

C.
Frequency

D.
Trade name of the drug

1 points
Question 9
A patient has been prescribed a Scheduled 5 drug, an example of this drug is

A.
Antianxiety

B.
Narcotic Analgesics

C.
Barbituates

D.
Antitussives

1 points
Question 10
A nurse is providing a patient with a list of drugs as a part of the patient’s plan of care. Which of the following drug nomenclatures should the nurse use to list the drugs?

A.
The drugs’ biologic names

B.
The drugs’ chemical names

C.
The drugs’ trade names

D.
The drugs’ generic names

1 points
Question 11
These drugs have a potential for abuse, but the potential is lower than for drugs on Schedule 2. These drugs contain a combination of controlled and noncontrolled substances. Use of these drugs can cause a moderate to low physiologic dependence and a higher psychological dependence. A verbal order can be given to the pharmacy and the prescription can be refilled up to five times within 6 months. Examples include certain narcotics (codeine) and nonbarbiturate sedatives. This is scheduled drug

A.
Schedule 1

B.
Schedule 2

C.
Schedule 3

D.
Schedule 4

1 points
Question 12
A nurse is a member of a research team that is exploring unique differences in responses to drugs that each individual possesses, based on genetic make-up. This area of study is called

A.
pharmacotherapeutics.

B.
pharmacoeconomics.

C.
pharmacodynamics.

D.
pharmacogenomics.

1 points
Question 13
A nurse working for a drug company is involved in phase III drug evaluation studies. Which of the following might the nurse be responsible for during this stage of drug development?

A.
Monitoring drug effects in patients who are selected to participate in a study, who have the disease that the drug is meant to treat

B.
Administering investigational drugs to patients

C.
Informing healthy, young volunteer participants of possible risks that could occur from taking an experimental drug

D.
Working with animals who are given experimental drugs

1 points
Question 14
A patient will begin three new medications as part of her treatment plan. The nurse practitioner understands that proper disposal of medications is key when the nurse practitioner states

A.
Many medications can be potentially harmful if taken by someone other than the person the medication was prescribed.

B.
Consult with several pharmacists to determine the cost of each drug.

C.
educate the patient to discard syringes in recycle bin

D.
consult with the physician concerning choice of medications for the patient.

1 points
Question 15
The nurse practitioner orders Amoxicillin 500 mg tid? What is the total amount of medication patient will take per day?

A.
500 mg

B.
1000 mg

C.
1500 mg

D.
150 mg

1 points
Question 16
Drugs have a valid medical use but a high potential for abuse, both psychological and physiologic. In an emergency, a Schedule 2 drug may be prescribed by telephone if a written prescription cannot be provided at the time. However, a written prescription must be provided within 72 hours with the words authorization for emergency dispensing written on the prescription. These prescriptions cannot be refilled. A new prescription must be written each time. Examples include certain amphetamines and barbiturates. This is scheduled drug

A.
Schedule 1

B.
Schedule 2

C.
Schedule 3

D.
Schedule 4

1 points
Question 17
A nurse receives an order to administer a critically ill patient two drugs immediately (stat). The nurse begins the process by

A.
consulting a drug guide for compatibility.

B.
identifying the patient by checking the armband and asking the patient to state his name.

C.
questioning the patient concerning allergies.

D.
washing his or her hands before handling the medications.

1 points
Question 18
Which of the following serves to protect the public by ensuring the purity of a drug and its contents?

A.
Federal legislation

B.
United States Adopted Names Council

C.
Nursing drug guides

D.
American Pharmaceutical Association

1 points
Question 19
A Native American man who lives a traditional lifestyle is scheduled to have heart surgery. The tribal chief has requested that the tribe’s medicine man perform a ritual before the patient goes to surgery. The nurse’s response to this request should be

A.
“I don’t think that will help, but you have the right to perform the ritual.”

B.
“I’m sorry, but that will not be possible.”

C.
“I will need to inform the doctor and see if he will allow this.”

D.
“As long as the ritual does not interfere with the patient’s preoperative care, it will be okay.”

1 points
Question 20
A nurse is admitting a Mexican woman to the hospital who cannot speak or understand English. The patient is alone, and there is no interpreter available. When trying to communicate with the patient the nurse will

A.
use nonverbal language.

B.
exaggerate her mouth movements.

C.
speak loudly.

D.
seak fast.

1 points
Question 21
A physician has ordered subcutaneous injections of morphine, a narcotic, every 4 hours as needed for pain for a motor vehicle accident victim. The nurse is aware that there is a high abuse potential for this drug and that it is categorized as a

A.
C-IV drug.

B.
C-II drug.

C.
C-III drug.

D.
C-I drug.

1 points
Question 22
A nurse who provides care in a long-term care facility is documenting a new resident’s medication regimen on the resident’s intake admission. The nurse is documenting the generic, rather than proprietary, names of the resident’s current drugs because

A.
documentation of trade names for drugs requires permission from the manufacturer.

B.
there is a potential for confusion and miscommunication if proprietary names are used.

C.
drugs with generic names are prescribed and used differently than proprietary drugs.

D.
proprietary names of drugs can vary from state to state.

1 points
Question 23
A patient comes to primary clinic for strep throat. A throat swab culture is sent to lab. What information is required for the nurse practitioner to disclose on lab transmittal?

A.
National Provider Identification (NPI)

B.
Centers for Medicare & Medicaid Services (CMS)

C.
Health Insurance Portability and Accountability Act of 1996 (HIPPA)

D.
None of the above

1 points
Question 24
A 46-year-old white American has been prescribed a drug that binds to acid glycoproteins. The nurse understands that white Americans usually receive

A.
the recommended normal dose of drugs that bind to acid glycoproteins.

B.
a higher-than-normal dose of drugs that bind to acid glycoproteins.

C.
a lower-than-normal dose of drugs that bind to acid glycoproteins.

D.
one half the recommended dose of drugs that bind to acid glycoproteins.

1 points
Question 25
Drugs have a high potential for abuse. There is no routine therapeutic use for these drugs and they are not available for regular use. They may be obtained for “investigational use only” by applying to the U.S. Drug Enforcement Agency. Examples include heroin and LSD. Which scheduled drug is this?

A.
Schedule 1

B.
Schedule 2

C.
Schedule 3

D.
Schedule 4

NURS 6521 week 3 Assignments Essay

ORDER   HERE

NURS 6521N Week 3 Quiz (2018): Advanced Pharmacology: Walden University Question 1 A normal maintenance dose for digoxin is 0.125 to 0.5 mg/day. In which of the following patients would the nurse most likely administer a lower-than-normal maintenance dose of digoxin? A. A 32-year-old female with cardiomegaly B. A 25-year-old male with congestive heart failure and atrial fibrillation C. A 79-year-old male with cardiomegaly D. A 42-year-old female with a third heart sound 1 points Question 2 A nurse has been following a male patient who is taking hydralazine, clonidine, and a diuretic for hypertension. After 2 weeks of medication therapy, the patient begins to complain of numbness and tingling in his hands and feet. The nurse suspects that these signs or symptoms are A. signs of peripheral neuritis. B. an adverse effect of the diuretic. C. an idiosyncratic reaction to clonidine. D. a result of new onset diabetes. 1 points Question 3 Which of the following patients is most likely to benefit from the administration of an adrenergic agonist? A. A woman who has been admitted with a suspected ischemic stroke B. A woman who is in labor and may require a caesarean section C. A man who has a diagnosis of primary hypertension D. A man who is in cardiogenic shock following a myocardial infarction 1 points Question 4 A patient is diagnosed with heart failure and is started on digoxin. On the second day of therapy, lab values, the ECG results, and the patient’s symptoms indicate that the medication is not producing a therapeutic effect. An appropriate nursing action would be to A. seek orders to increase the dosage. B. continue to administer the drug as ordered. C. insist that another drug should be ordered in addition to the digoxin. D. seek orders to discontinue digoxin and start another drug. 1 points Question 5 A patient is brought to the emergency department in hypertensive crisis. Nitroprusside is administered intravenously. The patient experiences diaphoresis and dizziness. Which of the following is the appropriate action by the nurse? A. Give the nitroprusside at the ordered rate B. Ask the prescriber to reduce the dose C. Obtain an order for naloxone D. Slow the rate of the infusion 1 points Question 6 A nurse is caring for a male patient who has a diagnosis of coronary artery disease (CAD). His drug therapy includes lovastatin. Because the patient has a history of severe renal disease, the nurse will assess for which of the following? A. Increase in the statin tolerance level B. Decrease in plasma concentration of lovastatin C. Increase in plasma concentration of lovastatin D. Decrease in LDL 1 points Question 7 A 55-year-old man’s hypertension has not responded adequately to his current medication regimen consisting or an ACE inhibitor, a beta-blocker and a thiazide diuretic. As a result, he will soon begin taking hydralazine (Apresoline) in addition to his existing antihypertensives. The addition of this medication to his regimen means that the nurse must prioritize which of the following nursing actions? A. Close monitoring of the patient’s electrolyte levels B. Supervision of the patient when performing activities of daily living C. Checking the patient’s blood glucose levels before each meal and at bedtime D. Frequent assessment of the patient’s cardiac status 1 points Question 8 A nurse who provides care in a busy clinic is aware of the high incidence and prevalence of hyperlipidemia and the consequent need for antihyperlipidemics in many patients. Treatment of high cholesterol using statins would be contraindicated in which of the following patients? A. A female client who had a laparoscopic cholecystectomy (gall bladder removal) earlier this year B. A resident of a long-term care facility whose Alzheimer disease is being treated with donepezil (Aricept) C. An obese male client who is a heavy alcohol user and who has cirrhosis of the liver D. A 72-year-old man who has emphysema and a 55-pack-year history of cigarette smoking 1 points Question 9 A patient with class-IV CHF has a medication regimen consisting of metoprolol (Lopressor), enalapril (Vasotec), and furosemide (Lasix). In addition to regularly assessing the patient s heart rate, the nurse should prioritize assessment of the patient’s A. exercise tolerance. B. blood pressure. C. cognition. D. intake and output. 1 points Question 10 A nurse is providing patient education to a 35-year-old man who has been prescribed clonidine (Catapres) as part of step 2 antihypertensive therapy. The nurse should anticipate that the drug will be administered A. subcutaneously. B. orally. C. transdermally. D. intravenously. 1 points Question 11 A nurse is the cardiac care unit is preparing to hang an intravenous dose of dofetilide (Tikosyn) for a patient who has just been admitted. What is the most likely goal of this intervention? A. To convert the patient’s atrial fibrillation to normal sinus rhythm B. To treat sinus bradycardia accompanied by hypotension C. To treat the patient’s ventricular tachycardia D. To resolve the patient’s uncompensated heart failure 1 points Question 12 A patient is admitted to the emergency department with severe chest pain. The emergency department physician orders intravenous nitroglycerin 5 mcg/min, titrate dose by 5 mcg/min every 3 to 5 minutes per infusion pump as needed. Before administering the nitroglycerin, the nurse should prioritize which of the following assessments? A. Blood urea nitrogen (BUN) B. Urinary output C. Blood pressure D. Heart rate 1 points Question 13 A 77-year-old man’s chronic heart failure is being treated with a regimen of quinapril (Accupril) and furosemide (Lasix). Which of the following assessment findings would suggest that the loop diuretic is contributing to a therapeutic effect? A. The man’s chest sounds are clear and his ankle edema is lessened. B. The man’s glomerular filtration rate and creatinine levels are within reference ranges. C. The man’s potassium and sodium levels remain with reference ranges. D. The man’s heart rate is between 60 and 70 beats per minute with a regular rhythm. 1 points Question 14 An expected outcome for a patient who has just taken sublingual nitroglycerin should be A. increased heart rate and decreased blood pressure. B. decreased heart rate and increased blood pressure. C. increased heart rate and increased blood pressure. D. decreased heart rate and decreased blood pressure. 1 points Question 15 A nurse is responsible for patient education for a 48-year-old woman who has been prescribed colestipol. The nurse will instruct the patient to take the medication A. two hours before a meal. B. after a meal. C. at bedtime. D. before a meal. 1 points Question 16 A priority nursing assessment for a patient who is to receive an alpha- or beta-adrenergic antagonist would be to A. measure urine output. B. assess heart rate. C. check blood glucose level. D. monitor respiratory rate. 1 points Question 17 A nurse is caring for a patient who is admitted into the cardiac care unit with acute, decompensated heart failure. Nesiritide (Natrecor) has been ordered. When preparing for administration of the drug, the nurse will A. administer the initial IV bolus over approximately 60 seconds. B. add the reconstituted vial of medication to a 1,000 mL IV bag. C. shake the vial vigorously to mix the medication for reconstitution. D. use the reconstituted solution within a 36-hour period. 1 points Question 18 A resident of a long-term care facility receives 12.5 mg metoprolol (Lopressor) at 8 AM and 8 PM daily. Before administering this drug, the nurse should perform and document what assessments? A. Level of consciousness and pain level B. Oxygen saturation and respiratory rate C. Temperature and respiratory rate D. Heart rate and blood pressure 1 points Question 19 A 62-year-old man is admitted to the hospital with a diagnosis of chest pain. He has an order for 0.3 mg of sublingual nitroglycerin prn for chest pain. Which of the following actions should the nurse do first when he complains of chest pain? A. Ask the patient to lie back and try to relax B. Administer a tablet under his tongue and repeat the action in 5 and 10 minutes if the pain has not subsided C. Call the physician D. Have the patient swallow a tablet every 5 minutes for 15 minutes 1 points Question 20 A nurse explains to a patient that nitroglycerin patches should be applied in the morning and removed in the evening. This medication schedule reduces the potential for A. toxic effects. B. adverse effects. C. nitrate tolerance. D. nitrate dependence. 1 points Question 21 Several months of treatment with a statin accompanied by lifestyle modifications have failed to appreciably improve a patient’s cholesterol levels. Consequently, the patient has been prescribed cholestyramine. The nurse should recognize that this drugs achieves its therapeutic effect by A. binding LDL to HDL. B. inhibiting the synthesis of cholesterol. C. oxidizing cholesterol to bile acids. D. mitigating the harmful effects of atherosclerosis by promoting vasodilation. 1 points Question 22 A 77-year-old patient has a long-standing history of hypertension, a health problem that is being treated with metoprolol and a thiazide diuretic. Before administering the 8 AM dose of these medications, what assessments should the nurse perform and document? (Select all that apply.) A. Heart rate B. Oxygen saturation C. Blood pressure D. Respiratory rate E. Temperature 1 points Question 23 The nurse is performing patient education for a woman who will soon begin treatment of hyperlipidemia with simvastatin (Zocor). The patient has asked the nurse if there are any “bad side effects” that she should be aware of. Which of the following statements should underlie the nurse’s response? A. The patient may experience a dry cough, especially at night, for the first few weeks after starting to take the simvastatin. B. Most patients tolerate statins well, with minor muscle aches being among the most common adverse effects. C. The patient will have to schedule regular blood work to closely monitor her kidney function for the duration of treatment. D. Many patients experience gastrointestinal upset with statins, which can be partially alleviated by taking the drug with food. 1 points Question 24 A nurse has administered a medication to a patient with hypertension. The prescribed drug is supposed to decrease cardiac output. A decrease in cardiac output would most likely A. increase the synergistic effect of the drug. B. decrease the amount of circulating blood sent through the kidneys. C. increase the serum creatinine level. D. decrease drug absorption in the blood. 1 points Question 25 A clinic nurse has been assigned a 49-year-old female patient who has a history of diabetes. A recent diagnosis of hypertension has been made, and the patient has been prescribed a thiazide diuretic and labetalol. The patient will be scheduled to return to the clinic once a month for the next 6 months. A priority action by the nurse will be to A. monitor the patient’s respiratory rate. B. weigh the patient. C. monitor the patient’s blood pressure. D. question the patient about her dietary intake.

NURS 6521N Week 3 Quiz (2018): Advanced Pharmacology: Walden University Question 1 A normal maintenance dose for digoxin is 0.125 to 0.5 mg/day. In which of the following patients would the nurse most likely administer a lower-than-normal maintenance dose of digoxin? A. A 32-year-old female with cardiomegaly B. A 25-year-old male with congestive heart failure and atrial fibrillation C. A 79-year-old male with cardiomegaly D. A 42-year-old female with a third heart sound 1 points Question 2 A nurse has been following a male patient who is taking hydralazine, clonidine, and a diuretic for hypertension. After 2 weeks of medication therapy, the patient begins to complain of numbness and tingling in his hands and feet. The nurse suspects that these signs or symptoms are A. signs of peripheral neuritis. B. an adverse effect of the diuretic. C. an idiosyncratic reaction to clonidine. D. a result of new onset diabetes. 1 points Question 3 Which of the following patients is most likely to benefit from the administration of an adrenergic agonist? A. A woman who has been admitted with a suspected ischemic stroke B. A woman who is in labor and may require a caesarean section C. A man who has a diagnosis of primary hypertension D. A man who is in cardiogenic shock following a myocardial infarction 1 points Question 4 A patient is diagnosed with heart failure and is started on digoxin. On the second day of therapy, lab values, the ECG results, and the patient’s symptoms indicate that the medication is not producing a therapeutic effect. An appropriate nursing action would be to A. seek orders to increase the dosage. B. continue to administer the drug as ordered. C. insist that another drug should be ordered in addition to the digoxin. D. seek orders to discontinue digoxin and start another drug. 1 points Question 5 A patient is brought to the emergency department in hypertensive crisis. Nitroprusside is administered intravenously. The patient experiences diaphoresis and dizziness. Which of the following is the appropriate action by the nurse? A. Give the nitroprusside at the ordered rate B. Ask the prescriber to reduce the dose C. Obtain an order for naloxone D. Slow the rate of the infusion 1 points Question 6 A nurse is caring for a male patient who has a diagnosis of coronary artery disease (CAD). His drug therapy includes lovastatin. Because the patient has a history of severe renal disease, the nurse will assess for which of the following? A. Increase in the statin tolerance level B. Decrease in plasma concentration of lovastatin C. Increase in plasma concentration of lovastatin D. Decrease in LDL 1 points Question 7 A 55-year-old man’s hypertension has not responded adequately to his current medication regimen consisting or an ACE inhibitor, a beta-blocker and a thiazide diuretic. As a result, he will soon begin taking hydralazine (Apresoline) in addition to his existing antihypertensives. The addition of this medication to his regimen means that the nurse must prioritize which of the following nursing actions? A. Close monitoring of the patient’s electrolyte levels B. Supervision of the patient when performing activities of daily living C. Checking the patient’s blood glucose levels before each meal and at bedtime D. Frequent assessment of the patient’s cardiac status 1 points Question 8 A nurse who provides care in a busy clinic is aware of the high incidence and prevalence of hyperlipidemia and the consequent need for antihyperlipidemics in many patients. Treatment of high cholesterol using statins would be contraindicated in which of the following patients? A. A female client who had a laparoscopic cholecystectomy (gall bladder removal) earlier this year B. A resident of a long-term care facility whose Alzheimer disease is being treated with donepezil (Aricept) C. An obese male client who is a heavy alcohol user and who has cirrhosis of the liver D. A 72-year-old man who has emphysema and a 55-pack-year history of cigarette smoking 1 points Question 9 A patient with class-IV CHF has a medication regimen consisting of metoprolol (Lopressor), enalapril (Vasotec), and furosemide (Lasix). In addition to regularly assessing the patient s heart rate, the nurse should prioritize assessment of the patient’s A. exercise tolerance. B. blood pressure. C. cognition. D. intake and output. 1 points Question 10 A nurse is providing patient education to a 35-year-old man who has been prescribed clonidine (Catapres) as part of step 2 antihypertensive therapy. The nurse should anticipate that the drug will be administered A. subcutaneously. B. orally. C. transdermally. D. intravenously. 1 points Question 11 A nurse is the cardiac care unit is preparing to hang an intravenous dose of dofetilide (Tikosyn) for a patient who has just been admitted. What is the most likely goal of this intervention? A. To convert the patient’s atrial fibrillation to normal sinus rhythm B. To treat sinus bradycardia accompanied by hypotension C. To treat the patient’s ventricular tachycardia D. To resolve the patient’s uncompensated heart failure 1 points Question 12 A patient is admitted to the emergency department with severe chest pain. The emergency department physician orders intravenous nitroglycerin 5 mcg/min, titrate dose by 5 mcg/min every 3 to 5 minutes per infusion pump as needed. Before administering the nitroglycerin, the nurse should prioritize which of the following assessments? A. Blood urea nitrogen (BUN) B. Urinary output C. Blood pressure D. Heart rate 1 points Question 13 A 77-year-old man’s chronic heart failure is being treated with a regimen of quinapril (Accupril) and furosemide (Lasix). Which of the following assessment findings would suggest that the loop diuretic is contributing to a therapeutic effect? A. The man’s chest sounds are clear and his ankle edema is lessened. B. The man’s glomerular filtration rate and creatinine levels are within reference ranges. C. The man’s potassium and sodium levels remain with reference ranges. D. The man’s heart rate is between 60 and 70 beats per minute with a regular rhythm. 1 points Question 14 An expected outcome for a patient who has just taken sublingual nitroglycerin should be A. increased heart rate and decreased blood pressure. B. decreased heart rate and increased blood pressure. C. increased heart rate and increased blood pressure. D. decreased heart rate and decreased blood pressure. 1 points Question 15 A nurse is responsible for patient education for a 48-year-old woman who has been prescribed colestipol. The nurse will instruct the patient to take the medication A. two hours before a meal. B. after a meal. C. at bedtime. D. before a meal. 1 points Question 16 A priority nursing assessment for a patient who is to receive an alpha- or beta-adrenergic antagonist would be to A. measure urine output. B. assess heart rate. C. check blood glucose level. D. monitor respiratory rate. 1 points Question 17 A nurse is caring for a patient who is admitted into the cardiac care unit with acute, decompensated heart failure. Nesiritide (Natrecor) has been ordered. When preparing for administration of the drug, the nurse will A. administer the initial IV bolus over approximately 60 seconds. B. add the reconstituted vial of medication to a 1,000 mL IV bag. C. shake the vial vigorously to mix the medication for reconstitution. D. use the reconstituted solution within a 36-hour period. 1 points Question 18 A resident of a long-term care facility receives 12.5 mg metoprolol (Lopressor) at 8 AM and 8 PM daily. Before administering this drug, the nurse should perform and document what assessments? A. Level of consciousness and pain level B. Oxygen saturation and respiratory rate C. Temperature and respiratory rate D. Heart rate and blood pressure 1 points Question 19 A 62-year-old man is admitted to the hospital with a diagnosis of chest pain. He has an order for 0.3 mg of sublingual nitroglycerin prn for chest pain. Which of the following actions should the nurse do first when he complains of chest pain? A. Ask the patient to lie back and try to relax B. Administer a tablet under his tongue and repeat the action in 5 and 10 minutes if the pain has not subsided C. Call the physician D. Have the patient swallow a tablet every 5 minutes for 15 minutes 1 points Question 20 A nurse explains to a patient that nitroglycerin patches should be applied in the morning and removed in the evening. This medication schedule reduces the potential for A. toxic effects. B. adverse effects. C. nitrate tolerance. D. nitrate dependence. 1 points Question 21 Several months of treatment with a statin accompanied by lifestyle modifications have failed to appreciably improve a patient’s cholesterol levels. Consequently, the patient has been prescribed cholestyramine. The nurse should recognize that this drugs achieves its therapeutic effect by A. binding LDL to HDL. B. inhibiting the synthesis of cholesterol. C. oxidizing cholesterol to bile acids. D. mitigating the harmful effects of atherosclerosis by promoting vasodilation. 1 points Question 22 A 77-year-old patient has a long-standing history of hypertension, a health problem that is being treated with metoprolol and a thiazide diuretic. Before administering the 8 AM dose of these medications, what assessments should the nurse perform and document? (Select all that apply.) A. Heart rate B. Oxygen saturation C. Blood pressure D. Respiratory rate E. Temperature 1 points Question 23 The nurse is performing patient education for a woman who will soon begin treatment of hyperlipidemia with simvastatin (Zocor). The patient has asked the nurse if there are any “bad side effects” that she should be aware of. Which of the following statements should underlie the nurse’s response? A. The patient may experience a dry cough, especially at night, for the first few weeks after starting to take the simvastatin. B. Most patients tolerate statins well, with minor muscle aches being among the most common adverse effects. C. The patient will have to schedule regular blood work to closely monitor her kidney function for the duration of treatment. D. Many patients experience gastrointestinal upset with statins, which can be partially alleviated by taking the drug with food. 1 points Question 24 A nurse has administered a medication to a patient with hypertension. The prescribed drug is supposed to decrease cardiac output. A decrease in cardiac output would most likely A. increase the synergistic effect of the drug. B. decrease the amount of circulating blood sent through the kidneys. C. increase the serum creatinine level. D. decrease drug absorption in the blood. 1 points Question 25 A clinic nurse has been assigned a 49-year-old female patient who has a history of diabetes. A recent diagnosis of hypertension has been made, and the patient has been prescribed a thiazide diuretic and labetalol. The patient will be scheduled to return to the clinic once a month for the next 6 months. A priority action by the nurse will be to A. monitor the patient’s respiratory rate. B. weigh the patient. C. monitor the patient’s blood pressure. D. question the patient about her dietary intake.

NURS 6521 week 4 Assignments Essay

NURS 6521N Week 4 Quiz (2018): Advanced Pharmacology: Walden University Question 1 A nurse is providing education to a 56-year-old man who is admitted to the emergency department with an acute asthma attack. The nurse’s initial assessment reveals that the patient has a history of pneumonia, drinks large quantities of coffee, and eats a high-calorie diet. Albuterol is prescribed for him. The important consideration when the nurse is preparing a teaching plan will be that the patient A. prefers a high-calorie diet. B. has a high coffee intake. C. is a 56-year-old man. D. has a history of pneumonia. 1 points Question 2 A man who smokes one pack of cigarettes daily has been diagnosed with chronic bronchitis. He has been prescribed theophylline by his family doctor. What effect will the patient’s smoking status have on the therapeutic use of theophylline? A. The patient may require higher doses of theophylline than a nonsmoker B. Theophylline may be contraindicated due to the risk of an anaphylactoid reaction C. The patient may require administration of an inhaled bronchodilator before each dose of theophylline D. The patient will require serial blood testing to ensure a safe serum level of theophylline 1 points Question 3 The parents of a 7-year-old boy who has just been diagnosed with allergic asthma are being taught about their son’s medication regimen by the nurse. The nurse is currently teaching the parent’s about the appropriate use of a “rescue drug” for acute exacerbations of their son’s asthma. What drug should the nurse suggests the parents to use in these situations? A. Beclomethasone B. Acetylcysteine C. Theophylline D. Albuterol 1 points Question 4 A nurse is assigned to a patient who is receiving IV aminophylline. The nurse is aware that the IV infusion rate should be A. no more than 25 mg/minute. B. not less than 15 mg/minute. C. no more than 100 mg/minute. D. not less than 10 mg/minute. 1 points Question 5 A 70-year-old woman has a complex medical and a current drug regimen that includes calcium and vitamin D supplements for osteoporosis, metformin (Glucophage) for type 2 diabetes, phenelzine (Nardil) for depression, and metoprolol (Lopressor) and furosemide (Lasix) for hypertension. The woman is requesting dextromethorphan for the treatment of a recurrent cough. What component of her drug regimen contraindicates the use of dextromethorphan? A. Phenelzine (Nardil) B. Calcium supplements C. Metformin (Glucophage) D. Metoprolol (Lopressor) 1 points Question 6 A patient is in the clinic for seasonal allergic rhinitis. Loratadine (Claritin) is prescribed. Which of the following statements will the nurse include when providing patient education concerning this drug? A. “You should take the medication twice a day.” B. “Determine if the drug is going to make you drowsy before you drive.” C. “Be sure to take this medication on a full stomach.” D. “This drug has to be taken every 4 to 6 hours.” 1 points Question 7 A 72-year-old man is prescribed theophylline for symptomatic relief of bronchial asthma. Which of the following findings would alert the nurse to the need for close monitoring? A. Need for additional bronchodilation B. Hypersensitivity to povidone C. Signs of an active lung infection D. Enlarged prostate gland 1 points Question 8 A college student has presented the campus medical clinic complaining of cold symptoms that he has been experiencing for the past 2 weeks. He tells the nurse that he has been taking OTC decongestants twice daily since the onset of the cold. What teaching point should the nurse provide to this student? A. “You can probably expect to get relief within the next few days or a week at the longest.” B. “You might find yourself more congested than ever as your body compensates for your use of these drugs.” C. “It’s best to take small doses of decongestants several times daily rather than two larger doses each day.” D. “Over-the-counter decongestants don’t normally work; you’ll likely need to get a prescription for something more effective.” 1 points Question 9 A female patient calls the clinic and reports that since she has been taking dextromethorphan (Robitussin), she has been extremely drowsy and dizzy. The nurse will question the patient about which of the following? A. The amount of tea or soft drinks she has during the day B. The number of cups of coffee she has during the day C. The amount of water she drinks every day D. The amount of grapefruit or orange juice she has during the day 1 points Question 10 A 53-year-old man has been treated for severe asthma for several years with prednisone. Recently, his physician initiated alternate-day therapy for him. The patient tells the nurse that he would rather take the medication every day to prevent confusion. Which of the following would be the best response by the nurse? A. “This schedule will enable you to lose weight.” B. “This schedule will decrease the cost of your medication.” C. “This schedule will be more convenient for you.” D. “This schedule allows rest periods so that adverse effects are decreased but the anti-inflammatory effects continue.” 1 points Question 11 A 54-year-old male patient with small-cell lung cancer is receiving etoposide. The nurse will carefully monitor for which of the following? A. Hypoglycemia B. Hypertension C. Tachycardia D. Hypotension 1 points Question 12 A patient with a diagnosis of nonsmall cell lung cancer is currently undergoing chemotherapy. At the encouragement of a family member, the patient has announced to the nurse his intention to complement this treatment with a regimen of herbal remedies. How should the nurse respond to this patient’s statement? A. “It’s important to discuss what herbs you’ll be taking with your doctor or the pharmacist to make sure there aren’t any interactions.” B. “It’s likely best to begin taking herbal remedies after you’ve completed this round of chemotherapy.” C. “Unfortunately, your current chemotherapy program will likely preclude the use of herbs.” D. “If that’s something in which you and your family see potential benefit, I’d encourage you to pursue it.” 1 points Question 13 Which of the following will a nurse inform the patient is one of the most common adverse effects of guaifenesin (Robitussin)? A. Increased blood glucose B. Urinary retention C. Gastrointestinal distress D. Increased blood pressure 1 points Question 14 A 72-year-old female patient is prescribed ipratropium aerosol for pulmonary emphysema. The nurse will instruct her to “test spray” the new metered-dose inhaler (MDI) three times before using it to prevent which of the following? A. Asymptomatic elevated hepatic enzymes levels B. Anaphylactoid reaction C. Symptomatic hepatitis with hyperbilirubinemia D. Paradoxic acute bronchospasm 1 points Question 15 A 47-year-old woman has been diagnosed with open-angle glaucoma. Pilocarpine drops are prescribed. The nurse’s assessment reveals that the patient has worn soft contact lenses for 15 years. The nurse will instruct the patient to A. stop wearing the contact lenses during the pilocarpine therapy. B. apply the contact lenses and wait 5 minutes before applying the drops. C. remove the contact lenses before applying the drops. D. apply the drops directly on the contact lenses. 1 points Question 16 A 57-year-old male patient who has been prescribed doxorubicin for small-cell lung cancer is advised by the nurse to avoid taking aspirin or drugs that contain aspirin because it may A. cause extravasation injury. B. cause acute nausea and vomiting. C. cause a radiation recall reaction. D. promote bleeding. 1 points Question 17 A 20-year-old woman calls the clinic at 5 PM and reports to the nurse that she forgot to take her morning dose of fexofenadine (Allegra). She usually takes her evening dose at 8 PM. She wants to know what she should do. The nurse will instruct her to A. skip the evening dose and start back in the morning. B. forget about the morning dose and take the evening dose. C. take the morning dose now and the evening dose at the regular time. D. double her evening dose. 1 points Question 18 A 68-year-old man complains of a chronic, nonproductive cough. He states that he has to have relief, that he has been coughing every 2 to 3 minutes, and he is worn out. Dextromethorphan is prescribed for him. Before he leaves the clinic he asks how long it will take for the medicine to work. The nurse will advise him that he should experience therapeutic effects in A. 60 minutes. B. 15 to 30 minutes. C. 5 minutes. D. 10 minutes. 1 points Question 19 A nurse is caring for a female patient who has developed atelectasis because of thick mucus secretions. The nurse is monitoring IV administration of acetylcysteine (Mucomyst) and notices that the patient’s face is flushed. The appropriate nursing action would be to A. remove any residual drug from the patient’s face. B. slow the administration of acetylcysteine and administer diphenhydramine if ordered. C. stop the administration of acetylcysteine. D. dilute the acetylcysteine solution. 1 points Question 20 A female patient has been taking prednisone for her asthma for 1 month. The nurse will teach her to gradually decrease her dose of prednisone to avoid A. gastrointestinal problems. B. adrenal insufficiency. C. menstrual irregularities. D. hypokalemia. 1 points Question 21 A 24-year-old factory worker has been prescribed guaifenesin for the first time. Which of the following will be a priority assessment by the nurse before the patient’s first dose? A. The amount of alcohol consumption B. The amount of fatty foods eaten daily C. The amount of salt intake D. The number of cigarettes smoked per day 1 points Question 22 A high school student was diagnosed with asthma when he was in elementary school and has become accustomed to carrying and using his “puffers”. In recent months, he has become more involved in sports and has developed a habit of administering albuterol up to 10 times daily. The nurse should teach the student that overuse of albuterol can lead to A. severe anticholinergic effects. B. permanent liver damage. C. community-acquired pneumonia. D. rebound bronchoconstriction. 1 points Question 23 A 25-year-old woman comes to the clinic because of a chronic, nonproductive cough. Assessment reveals that she has a history of asthma and dextromethorphan, (Robitussin) is prescribed for her. The nurse will question this order because A. serotonin syndrome may result. B. persistent coughing may develop. C. respiratory secretions may be thickened. D. secretions may be retained. 1 points Question 24 A nurse is providing patient education to a 42-year-old man who has been prescribed dextromethorphan (Robitussin). The patient is a crane operator for a local construction company. The nurse’s assessment reveals that the patient is a smoker and has diabetes. The most important safety consideration while the nurse is designing a plan of care for this patient would be the fact that he is A. a diabetic. B. 42 years of age. C. a crane operator. D. a smoker. 1 points Question 25 A patient with bronchial asthma is prescribed a sustained-release preparation of theophylline. To help minimize the adverse effects of the drug, which of the following should the nurse suggest? A. Take it with a meal B. Take it on an empty stomach, 1 hour before or 2 hours after meals C. Take it with foods containing xanthines, such as caffeine D. Increase the drug dosage up to 50% 1 points Question 26 A female patient, age 36, is prescribed inhaled corticosteroid (ICS) for daily use. Which of the following adverse effects should the nurse closely monitor for in this patient? A. Oropharyngeal Candida albicans infection B. Suppression of the hypothalamic-pituitary-adrenal axis C. Delayed growth D. Hypotension 1 points Question 27 After a recent history of shortness of breath that has become increasingly severe, a woman has been prescribed ipratropium by MDI while she undergoes a diagnostic workup. What patient teaching should the nurse provide to this patient? A. “Make sure that you take this at the scheduled times, regardless of whether you feel short of breath.” B. “It’s important that you perform deep breathing exercises before and after taking ipratropium.” C. “You should think of this as an ’emergency drug’, and it should only be used when you experience symptoms.” D. “Most people find that ipratropium cures their lung disease within a few months.” 1 points Question 28 Children age 6 to 11 are recommended to start with what dosage of Allegra? A. 30 mg twice a day B. 65 mg twice a day C. 120 mg twice a day D. 240 mg once a day 1 points Question 29 A male patient is to begin treatment for pneumonia with an albuterol (Ventolin) inhaler. The nurse will advise the patient that he will most likely experience which of the following common adverse effects of the drug? A. Headache B. Dyspepsia C. Muscle cramps D. Throat irritation 1 points Question 30 A woman has a long history of seasonal allergies that are typically accompanied by rhinorrhea, red eyes, and frequent sneezing. Sneezes are a result of A. stimulation of goblet cells and mast cells by the presence of microorganisms. B. elevated CO2 levels that are detected by chemoreceptors in the large vessels. C. a CNS reflex that is initiated by receptors in the nasal cavities. D. a reflex that is caused when irritants come into contact with cilia.

NURS 6521 week 6 Assignments Essay

NURS 6521N Week 6 Quiz (2018): Advanced Pharmacology: Walden University Question 1 1. Which of the following would be the most important safety-related instruction for a nurse to give to a patient taking baclofen (Lioresal)? A. Learn to self-administer the drug B. Do not walk or drive after taking the drug C. Discontinue the use of alcohol D. Take a laxative when needed 1 points Question 2 1. A clinic nurse is following a 9-year-old boy who is taking somatropin. Which of the following will the nurse monitor periodically? A. Antidiuretic hormone (ADH) levels B. Dehydration C. Height D. Water retention 1 points Question 3 1. A nurse is developing a care plan for a patient who has multiple sclerosis. An expected outcome for the patient who is receiving glatiramer would be a decrease in A. chest pain. B. heart palpitations. C. breathing difficulties. D. fatigue. 1 points Question 4 1. A patient with diabetes has had a cough for 1 week and has been prescribed a cough syrup (an expectorant). What special instructions should the nurse include in the patient teaching for this situation? A. Monitor glucose levels closely B. Wash hands before and after taking the medicine C. Note the time the medicine is taken each day D. Keep track of any gastrointestinal tract infections 1 points Question 5 1. A 70-year-old woman who is on long-term ibuprofen therapy for osteoarthritis has returned to the clinic for her regular 6-month visit. In the last couple of months, she has been having increasing periods of abdominal pain. The nurse suspects that this pain may be related to A. anemia. B. interstitial nephritis. C. peptic ulcer disease or gastritis. D. constipation. 1 points Question 6 1. A nurse is caring for a 61-year-old man who has had a severe attack of gout while in the hospital for food poisoning. The nurse administers colchicine intravenously in order to A. avoid aggravating the gastrointestinal tract. B. minimize the risk of depressed bone marrow function. C. ensure quick distribution of the drug. D. prevent the risk of infection or bleeding. 1 points Question 7 1. A patient receives 25 units of NPH insulin at 7.AM. At what time of day should the nurse advise the patient to be most alert for a potential hypoglycemic reaction? A. Before lunch B. Late afternoon C. After breakfast D. Bedtime 1 points Question 8 1. A nurse is teaching a patient about his newly prescribed drug, colchicine, for gout. The nurse will instruct the patient to avoid which of the following foods? A. Milk B. Shrimp C. Eggs D. Green beans 1 points Question 9 1. A 43-year-old woman was diagnosed with multiple sclerosis 2 years ago and has experienced a recent exacerbation of her symptoms, including muscle spasticity. Consequently, she has been prescribed Dantrolene (Dantrium). In light of this new addition to her drug regimen, what teaching point should the woman’s nurse provide? A. “This will likely relieve your muscle spasms but you’ll probably develop a certain amount of dependence on the drug over time.” B. “We’ll need to closely monitor your blood sugar levels for the next week.” C. “There’s a small risk that you might experience some hallucinations in the first few days that you begin taking this drug.” D. “You might find that this drug exacerbates some of your muscle weakness while it relieves your spasticity.” 1 points Question 10 1. A 13-year-old patient has juvenile arthritis. He has recently had oral surgery and was told by the surgeon to take aspirin for the pain. The nurse will monitor for which of the following? A. Agranulocytosis B. Hepatotoxicity C. Aplastic anemia D. Bronchoconstriction 1 points Question 11 1. A 33-year-old man has developed acute gouty arthritis. He has been prescribed colchicine. When developing a care plan for this patient, which factor will be most important for the nurse to consider? A. Ethnicity B. Typical daily fluid intake C. Dietary habits D. Work environment 1 points Question 12 1. A diabetic patient being treated for obesity tells the nurse that he is having adverse effects from his drug therapy. The patient has been taking dextroamphetamine for 2 weeks as adjunct therapy. Which of the following adverse effects would need the nurse’s immediate attention? A. Jittery feeling B. Decreased libido C. Dry eyes D. Increased blood glucose 1 points Question 13 1. A patient in need of myocardial infarction prophylaxis has been prescribed sulfinpyrazone for gout. Which of the following will the nurse monitor the patient most closely for? A. Renal dysfunction B. Hypothermia C. Bleeding D. Hypotension 1 points Question 14 1. A nurse is working with a 57-year-old man who is a former intravenous drug abuser. He has been prescribed a weekly dosage of methotrexate for his rheumatoid arthritis. Which of the following will the nurse include in her teaching plan for this patient? A. Avoid high-fat foods B. Drink plenty of water every day C. Avoid red meat D. Take the tablets before bedtime 1 points Question 15 1. During long-term desmopressin therapy in a 48-year-old woman, it will be most important for the nurse to assess which of the following? A. The patient’s diet B. The patient’s environment C. The condition of the patient’s skin D. The condition of the patient’s nasal passages 1 points Question 16 1. A patient is taking gabapentin (Neurontin) for spasticity associated with multiple sclerosis. Which of the following should be the priority for monitoring? A. Cardiac function B. Respiratory function C. Hepatic function D. Renal function 1 points Question 17 1. A nurse is instructing a patient in the administration of regular insulin by the subcutaneous route. Which of the following strategies would the nurse suggest if the goal is to promote absorption of the regular insulin? A. Rotate injection sites by using the arm one day, the stomach the next day, and the thigh the day after and then repeating the cycle. B. Administer the medication 30 to 60 minutes after a meal. C. Select one injection site for regular insulin injections and use it exclusively. D. Select one anatomic area for regular insulin injections and then use serial locations within that area. 1 points Question 18 1. A nurse will instruct a patient taking allopurinol to take each dose A. at night. B. after a meal. C. first thing in the morning. D. before a meal. 1 points Question 19 1. A male patient with a diagnosis of relapsing-remitting multiple sclerosis is in the clinic to discuss with the nurse the possibility of self-administration of glatiramer. During the patient education session for self-administration, the nurse will emphasize A. the need to use only the thigh muscle for the drug injection site. B. the need to avoid crushing the tablet. C. the need to rotate the injection site of the drug. D. the need to place the tablet under the tongue. 1 points Question 20 1. The nurse is conducting a medication reconciliation of a new resident of a long-term care facility. The nurse notes that the resident takes allopurinol on a daily basis for the treatment of gout. What is the primary purpose of this drug? A. To balance urate concentration and prevent gout attacks B. To promote the remodeling of damaged synovium C. To potentiate the metabolism of dietary purines D. To achieve pain relief in joints affected by gout 1 points Question 21 1. A 65-year-old woman has an advanced form of rheumatoid arthritis. Her treatment includes a regular dosage of methotrexate. The nurse will advise her to take which of the following vitamin supplements while taking the drug? A. Vitamin C B. Vitamin B C. Vitamin D D. Vitamin A 1 points Question 22 1. A 66-year-old woman has experienced a significant decline in her quality of life as a result of worsening rheumatoid arthritis. Her physician has prescribed etanercept and the nurse is responsible for facilitating this new aspect of the patient’s drug regimen. This will involve the administration of A. weekly administration of an enteric coated tablet. B. weekly subcutaneous injections. C. daily, oral administration . D. daily, outpatient intravenous therapy. 1 points Question 23 1. A nurse has been invited to speak to a support group for persons with movement disorders and their families. Which of the following statements by the nurse addresses the chronic nature of these diseases and the relevant drug therapies? A. “Drugs do not cure these disorders; they instead enhance quality of life.” B. “Drugs used to treat these disorders always pose a risk of severe liver and kidney dysfunction.” C. “Drug therapy can consist of one or more drugs to eliminate the symptoms of these diseases.” D. “Persons of all cultures are treated similarly and respond in similar ways to treatment.” 1 points Question 24 1. A nurse is caring for a patient who has been diagnosed with hypothyroidism. Levothyroxine (Synthroid) has been prescribed. Before the drug therapy is started, the nurse will assess for which of the following? A. The patient’s age B. Allergy to seafood C. History of taking anticoagulant drugs D. Hirsutism 1 points Question 25 1. A nurse will monitor which of the following when assigned to a patient taking allopurinol for chronic gout who visits the clinic every 2 months? A. Liver function tests B. Blood pressure C. Blood sugar levels D. Body temperature 1 points Question 26 1. To minimize the risk of adverse effects of glucagon when given to an unconscious diabetic patient, as the patient regains consciousness, the nurse should A. administer carbohydrates. B. monitor for nausea and vomiting. C. administer calcium supplements. D. position the patient in the side-lying position. 1 points Question 27 1. A nurse is instructing a patient who was recently diagnosed with multiple sclerosis about dantrolene (Dantrium). The patient is a 38-year-old-male and the foreman for a construction company. In order to minimize one important adverse effect of the drug, the nurse will give the patient which of the following instructions? A. Decrease the dosage if any adverse effect is experienced B. Have a complete blood cell count done weekly C. Wear appropriate clothing and sunscreen whenever he is in direct sunlight D. Eat a high-protein diet 1 points Question 28 1. A 32-year-old female patient is taking tizanidine (Zanaflex) for spasticity related to her multiple sclerosis. The nurse will inform the patient and her husband that the adverse effect that poses the greatest safety risk to the patient is A. constipation. B. dry mouth. C. fatigue. D. hypotension. 1 points Question 29 1. Following an assessment by her primary care provider, a 70-year-old resident of an assisted living facility has begun taking daily oral doses of levothyroxine. Which of the following assessment findings should prompt the nurse to withhold a scheduled dose of levothyroxine? A. The resident has not eaten breakfast because of a recent loss of appetite B. The resident’s apical heart rate is 112 beats/minute with a regular rhythm C. The resident received her annual influenza vaccination the previous day D. The resident had a fall during the night while transferring from her bed to her bathroom 1 points Question 30 1. A 34-year-old male patient is prescribed methimazole (MMI). The nurse will advise him to report which of the following immediately? A. Epigastric distress B. Vertigo C. Intolerance to cold D. Loss of appetite

NURS 6521 week 7 Assignments Essay

Pharmacotherapy for Gastrointestinal and Hepatobiliary Disorders

Gastrointestinal (GI) and hepatobiliary disorders affect the structure and function of the GI tract. Many of these disorders often have similar symptoms such as abdominal pain, cramping, constipation, nausea, bloating, and fatigue. Since multiple disorders can be tied to the same symptoms, it is important for advanced practice nurses to carefully evaluate patients and prescribe treatment that targets the cause rather than the symptom. Once the underlying cause is identified, an appropriate drug therapy plan can be recommended based on medical history and individual patient factors. In this Discussion, you examine a case study of a patient who presents with symptoms of a possible GI/hepatobiliary disorder, and you design an appropriate drug therapy plan.

Consider the following case study:

Patient HL comes into the clinic with the following symptoms: nausea, vomiting, and diarrhea. The patient has a history of drug abuse and possible Hepatitis C. HL is currently taking the following prescription drugs:

Synthroid 100 mcg daily
Nifedipine 30 mg daily
Prednisone 10 mg daily

To prepare:

· Review this week’s media presentation on pharmacology for the gastrointestinal system.

· Review the provided case study. Reflect on the patient’s symptoms, medical history, and drugs currently prescribed.

· Think about a possible diagnosis for the patient. Consider whether the patient has a disorder related to the gastrointestinal and hepatobiliary system or whether the symptoms are the result of a disorder from another system or other factors such as pregnancy, drugs, or a psychological disorder.

· Consider an appropriate drug therapy plan based on the patient’s history, diagnosis, and drugs currently prescribed.

Post 1 page page APA format

an explanation of your diagnosis for the patient including your rationale for the diagnosis.

Then, describe an appropriate drug therapy plan based on the patient’s history, diagnosis, and drugs currently prescribed. (Note this is a pharmacology class make sure medicine is appropriate and will work with pt’s current med list)

Readings
· Arcangelo, V. P., & Peterson, A. M. (Eds.). (2013). Pharmacotherapeutics for advanced practice: A practical approach (3rd ed.). Ambler, PA: Lippincott Williams & Wilkins.

o Chapter 28, “Nausea and Vomiting” (pp. 392–409)

This chapter focuses on the etiology of nausea and vomiting, as well as body systems that impact or trigger nausea and vomiting. It also covers various drugs used to treat nausea and vomiting including the process of selecting, administering, and managing drug therapy for patients.

o Chapter 29, “Gastroesophageal Reflux Disease and Peptic Ulcer Disease” (pp. 410–423)

This chapter begins with an overview of risk factors, symptoms, and clinical stages of gastroesophageal reflux disease (GERD). It then examines drugs used to treat GERD and peptic ulcer disease (PUD), including proper dosages, possible adverse reactions, contraindications, and special considerations.

o Chapter 30, “Constipation, Diarrhea, and Irritable Bowel Syndrome” (pp. 424–453)

This chapter begins by exploring disorders associated with constipation and diarrhea, as well as drugs used in treatment. It also covers the pathophysiology of irritable bowel syndrome and related drug therapies.

o Chapter 31, “Inflammatory Bowel Disease” (pp. 454–471)

This chapter examines the causes, pathophysiology, and diagnostic criteria of inflammatory bowel disease (IBD), Crohn’s disease, and ulcerative colitis. It also identifies drugs used to treat IBD, including proper dosage, adverse reactions, and special considerations.

· Starr, S., & Raines, D. (2011). Cirrhosis: Diagnosis, management, and prevention. American Family Physician, 84(12), 1353–1359.
Retrieved from the Walden Library databases.

This article examines the clinical presentation, pathophysiology, diagnosis, management, and prevention of cirrhosis. It also provides methods for managing complications of cirrhosis.

· Drugs.com. (2012). Retrieved from http://www.drugs.com/

This website presents a comprehensive review of prescription and over-the-counter drugs including information on common uses and potential side effects. It also provides updates relating to new drugs on the market, support from health professionals, and a drug-drug interactions checker.

NURS 6521 week 8 Assignments Essay

Week 8: Infections

Prior to the H1N1 influenza pandemic in 2009, it had been more than 40 years since an infectious virus had resulted in such substantial illness, hospitalizations, and deaths on a global scale (CDC, 2010). This can be attributed to improved prevention such as proper hygiene, as well as vaccinations and other drug therapies. However, as the H1N1 pandemic illustrates, infections are sometimes unavoidable even when appropriate prevention methods are implemented. In clinical settings, patients present with various infections including common disorders such as flus and colds, as well as disorders that require more extensive treatment and care such as the human immunodeficiency virus (HIV) or tuberculosis (TB). As an advanced practice nurse, you must evaluate patients presenting with symptoms of infections and recommend appropriate drug treatments.

This week you begin to explore infections by considering issues surrounding the prevalence, management, and education about HIV/AIDS. You also examine viral and bacterial infections, as well as the appropriate use of antimicrobial agents.

Learning Objectives

By the end of this week, students will:

· Analyze reasons for the prevalence of HIV/AIDS

· Analyze ways health care professionals can change society’s perceptions of HIV/AIDS

· Analyze strategies to educate HIV positive patients on treatment and management

· Analyze categories of antimicrobial agents

· Differentiate between viral and bacterial infections

· Analyze the relationship between infections and antimicrobial agents

· Understand and apply key terms, concepts, and principles related to prescribing drugs to treat infections and infestations

Photo Credit: E.M. Pasieka/ Science Photo Library/Getty Images

Learning Resources

This page contains the Learning Resources for this week. Be sure to scroll down the page to see all of this week’s assigned Learning Resources. To access select media resources, please use the media player below.

Required Readings

Note: To access this week’s required library resources, please click on the link to the Course Readings List, found in the Course Materials section of your Syllabus.

Arcangelo, V. P., & Peterson, A. M. (Eds.). (2013). Pharmacotherapeutics for advanced practice: A practical approach (3rd ed.). Ambler, PA: Lippincott Williams & Wilkins.

· Chapter 8, “Principles of Antimicrobial Therapy” (pp. 96–117) This chapter covers factors that impact the selection of an antimicrobial treatment regimen. It also examines the clinical uses, adverse events, and drug interactions of various antimicrobial agents such as penicillin.

· Chapter 12, “Fungal Infections of the Skin” (pp. 141–149) This chapter explores the pathophysiology of several fungal infections of the skin as well as related drug treatments and examines the importance of patient education when managing these infections.

· Chapter 14, “Bacterial Infections of the Skin” (pp. 158–172) This chapter begins by examining causes of bacterial infections. It then explores the importance of selecting an appropriate agent for treating bacterial infections.

· Chapter 32, “Urinary Tract Infection” (pp. 474–480) This chapter covers drugs used to treat urinary tract infections and identifies special considerations when treating geriatric patients, pediatric patients, and women.

· Chapter 35, “Sexually Transmitted Infections” (pp. 512–535) This chapter outlines the causes, pathophysiology, and drug treatment of six sexually transmitted infections, including gonorrhea, syphilis, and human papilloma virus infection (HPV). It also examines the importance of selecting the proper agent and monitoring patient response to treatment.

· Chapter 48, “Human Immunodeficiency Virus” (pp. 748–762) This chapter presents the causes, pathophysiology, diagnostic criteria, and prevention methods for HIV. It also covers various methods of drug treatment and patient factors to consider when selecting, administering, and managing drug treatments.

Krummenacher, I., Cavassini, M., Bugnon, O., & Schneider, M. (2011). An interdisciplinary HIV-adherence program combining motivational interviewing and electronic antiretroviral drug monitoring. AIDS Care, 23(5), 550–561.

Note: Retrieved from the Walden Library databases.

This article analyzes medication adherence in HIV patients and examines factors that increase adherence as well as factors that contribute to termination or discontinuation of treatment.

Drugs.com. (2012). Retrieved from http://www.drugs.com/

This website presents a comprehensive review of prescription and over-the-counter drugs including information on common uses and potential side effects. It also provides updates relating to new drugs on the market, support from health professionals, and a drug-drug interactions checker.

Scourfield, A., Waters, L., & Nelson, M. (2011). Drug combinations for HIV: What’s new? Expert Review of Anti-Infective Therapy, 9(11), 1001–1011.

Note: Retrieved from the Walden Library databases.

This article examines current therapies and strategies for treating HIV patients. It also examines factors that impact selection of therapy, including drug interactions, personalization of therapy, costs, management of comorbidities, and patient response.

Required Media

Laureate Education, Inc. (Executive Producer). (2012). Antimicrobials. Baltimore, MD: Author.

This media presentation outlines principles of antimicrobial therapy

Note: The approximate length of this media piece is 7 minutes.

Optional Resources

Mayer, K. H., & Krakower, D. (2012). Antiretroviral medication and HIV prevention: New steps forward and New Questions. Annals of Internal Medicine, 156(4), 312–314.

Note: Retrieved from the Walden Library databases.

Assignment: Antimicrobial Agents
Antimicrobial agents are essential components in the treatment of various bacterial infections as they help to kill or prevent the growth of microbes such as bacteria, fungi, and protozoans. Prior to the discovery of antimicrobial agents, treatment options for patients with bacterial infections were limited. For many patients, treatment often resulted in the amputation of limbs or even death. Today, treatment options for bacterial infections typically have a more positive prognosis. Due to the various types of infections presented in patients, it is essential to be able to identify the underlying cause of the infection—whether bacterial or viral—before recommending drug treatments. This will help you identify whether or not an antimicrobial agent would be appropriate and which specific agent would target the infection. In this Assignment, you consider the appropriate use of antimicrobial agents for infections.

To prepare:
· Review this week’s media presentation on principles of antimicrobial therapy, as well as Chapter 8 of the Arcangelo and Peterson text.

· Consider the categories of antimicrobial agents.

· Think about differences between viral and bacterial infections.

· Reflect on why proper identification of the infection is key to selecting the proper antimicrobial agent.

By Day 7
Write a 2- to 3- page paper that addresses the following:

· Describe the categories of antimicrobial agents.

· Describe differences between viral and bacterial infections.

· Explain why proper identification of viral and bacterial infections is key to selecting the proper antimicrobial agent.

Reminder: The School of Nursing requires that all papers submitted include a title page, introduction, summary, and references. The Sample Paper provided at the Walden Writing Center provides an example of those required elements (available at http://writingcenter.waldenu.edu/57.htm). All papers submitted must use this formatting.

Application: Antimicrobial Agents

Antimicrobial agents are essential components in the treatment of various bacterial infections as they help to kill or prevent the growth of microbes such as bacteria, fungi, and protozoans. Prior to the discovery of antimicrobial agents, treatment options for patients with bacterial infections were limited. For many patients, treatment often resulted in the amputation of limbs or even death. Today, treatment options for bacterial infections typically have a more positive prognosis. Due to the various types of infections presented in patients, it is essential to be able to identify the underlying cause of the infection—whether bacterial or viral—before recommending drug treatments. This will help you identify whether or not an antimicrobial agent would be appropriate and which specific agent would target the infection. In this Assignment, you consider the appropriate use of antimicrobial agents for infections.

To prepare:

· Review this week’s media presentation on principles of antimicrobial therapy, as well as Chapter 8 of the Arcangelo and Peterson text.

· Consider the categories of antimicrobial agents.

· Think about differences between viral and bacterial infections.

· Reflect on why proper identification of the infection is key to selecting the proper antimicrobial agent.

To complete:

Write a 2- to 3- page paper (APA format) that addresses the following:

1. Describe the categories of antimicrobial agents. (use generic names)

2. Describe differences between viral and bacterial infections.

3. Explain why proper identification of viral and bacterial infections is key to selecting the proper antimicrobial agent.

The School of Nursing requires that all papers submitted include a title page, introduction, summary, and references

Readings
· Arcangelo, V. P., & Peterson, A. M. (Eds.). (2013). Pharmacotherapeutics for advanced practice: A practical approach (3rd ed.). Ambler, PA: Lippincott Williams & Wilkins.

o Chapter 8, “Principles of Antimicrobial Therapy” (pp. 96–117)

This chapter covers factors that impact the selection of an antimicrobial treatment regimen. It also examines the clinical uses, adverse events, and drug interactions of various antimicrobial agents such as penicillin.

o Chapter 12, “Fungal Infections of the Skin” (pp. 141–149)

This chapter explores the pathophysiology of several fungal infections of the skin as well as related drug treatments and examines the importance of patient education when managing these infections.

o Chapter 14, “Bacterial Infections of the Skin” (pp. 158–172)

This chapter begins by examining causes of bacterial infections. It then explores the importance of selecting an appropriate agent for treating bacterial infections.

o Chapter 32, “Urinary Tract Infection” (pp. 474–480)

This chapter covers drugs used to treat urinary tract infections and identifies special considerations when treating geriatric patients, pediatric patients, and women.

o Chapter 35, “Sexually Transmitted Infections” (pp. 512–535)

This chapter outlines the causes, pathophysiology, and drug treatment of six sexually transmitted infections, including gonorrhea, syphilis, and human papilloma virus infection (HPV). It also examines the importance of selecting the proper agent and monitoring patient response to treatment.

o Chapter 48, “Human Immunodeficiency Virus” (pp. 748–762)

This chapter presents the causes, pathophysiology, diagnostic criteria, and prevention methods for HIV. It also covers various methods of drug treatment and patient factors to consider when selecting, administering, and managing drug treatments.

· Krummenacher, I., Cavassini, M., Bugnon, O., & Schneider, M. (2011). An interdisciplinary HIV-adherence program combining motivational interviewing and electronic antiretroviral drug monitoring.AIDS Care, 23(5), 550–561.
Retrieved from the Walden Library databases.

This article analyzes medication adherence in HIV patients and examines factors that increase adherence as well as factors that contribute to termination or discontinuation of treatment.

· Drugs.com. (2012). Retrieved from http://www.drugs.com/

This website presents a comprehensive review of prescription and over-the-counter drugs including information on common uses and potential side effects. It also provides updates relating to new drugs on the market, support from health professionals, and a drug-drug interactions checker.

· Scourfield, A., Waters, L., & Nelson, M. (2011). Drug combinations for HIV: What’s new? Expert Review of Anti-Infective Therapy, 9(11), 1001–1011. Retrieved from the Walden Library databases

This article examines current therapies and strategies for treating HIV patients. It also examines factors that impact selection of therapy, including drug interactions, personalization of therapy, costs, management of comorbidities, and patient response.

nurs 6521 week 9 assignment

NURS 6501N Week 9 Quiz Set 1 – Question and Answers
NURS 6501N Week 9 Quiz
1. While planning care for a patient with hypothyroidism, which principle should the nurse remember? The basal metabolic rate is unusually _____ with hypothyroidism.
A 50-year-old female presents with lightheadedness and overall abnormal feelings. Hyperaldosteronism is diagnosed. Which of the following symptoms would the nurse expect?
A patient wants to know why ADH is important in the body. What is the nurse’s best response? ADH is important in:
When catecholamines are released in a patient, what should the nurse assess for?
Visual disturbances are a common occurrence in patients with untreated Graves disease. The endocrinologist explains to the patient that the main cause of these complications is:
If the patient has a problem with the pineal gland, which substance would the nurse monitor?
A nurse recalls insulin has an effect on which of the following groups of electrolytes?
A 40-year-old male undergoes surgery for a PTH-secreting tumor in which the parathyroid is removed. Which of the following would the nurse expect following surgery?
If a patient’s posterior pituitary is removed, which hormone would the nurse expect to decrease?
A 45-year-old female has elevated thyroxine production. Which of the following would accompany this condition?
A 50-year-old male patient presents with polyuria and extreme thirst. He was given exogenous ADH. For which of the following conditions would this treatment be effective?
When a nurse is assessing the physical features of individuals with Cushing syndrome, these findings will include:
A 25-year-old male presents with fatigue, constipation, and sexual dysfunction. Tests reveal all pituitary hormones are normal and no masses are present. The nurse suspects the most likely cause of his symptoms is a dysfunction in the:
A nurse is teaching a patient about insulin. Which information should the nurse include? Insulin is primarily regulated by:
A 30-year-old male presents to his primary care provider reporting visual disturbances. CT reveals a pituitary tumor and lab tests reveal elevated prolactin. He is diagnosed with prolactinoma. Which of the following treatments would the nurse help implement?
A 22-year-old female has a low level of TSH. What condition does the nurse expect the patient is experiencing?
A 15-year-old female presents with breast discharge, dysmenorrhea, and excessive excitability. Tests reveal that all her pituitary hormones are elevated. What does the nurse suspect as the most likely cause for these assessment findings?
A nurse is caring for a patient with SIADH. What severe complication should the nurse assess for?
A student asks the instructor which of the following is the most potent naturally occurring glucocorticoid. How should the instructor respond?
If a patient had a problem with the hypothalamus, which of the following hormones would be affected?
A patient has high levels of hormones. To adapt to the high hormone concentrations, the patient’s target cells have the capacity for:
A 35-year-old female took corticosteroid therapy for several months. Which of the following would the nurse expect to find?
A 50-year-old male patient is deficient in ADH production. Which of the following assessment findings would the nurse expect to find?
A 54-year-old patient with pulmonary tuberculosis (lung infection) is evaluated for syndrome of inappropriate ADH secretion (SIADH). Which of the following electrolyte imbalances would be expected in this patient?
A 22-year-old male is admitted to the intensive care unit with a closed head injury sustained in a motorcycle accident. The injury has caused severe damage to the posterior pituitary. Which of the following complications should the nurse anticipate?
Which principle should the nurse include while planning care for a patient with an ADH problem? ADH release from the posterior pituitary is stimulated by:
A 30-year-old male was diagnosed with hypothyroidism. Synthesis of which of the following would decrease in this patient?
What common neurologic disturbances should the nurse assess for in a patient with a pituitary adenoma?
A 30-year-old male is diagnosed with a hormone-secreting tumor of the pancreas alpha cells. Which of the following would the nurse expect to be most likely increased in this patient?
An endocrinologist isolated a new hormone and found it to be a water-soluble amine. Which of the following is most like this new hormone?
A 25-year-old female with Graves disease is admitted to a medical-surgical unit. Palpation of her neck would most likely reveal:
A 19-year-old female with type 1 DM was admitted to the hospital with the following lab values: serum glucose 500 mg/dl (high), urine glucose and ketones 4+ (high), and arterial pH 7.20 (low). Her parents state that she has been sick with the “flu” for a week. Which of the following statements best explains her acidotic state?
A 3-year-old male was diagnosed with congenital hypothyroidism. The parents ask the nurse if left untreated what will happen. What is the nurse’s best response? If left untreated, the child would have:
A 35-year-old female with Graves disease is admitted to a medical-surgical unit. While the nurse is reviewing the lab tests, which results would the nurse expect to find?
A 12-year-old female is newly diagnosed with type 1 DM. When the parents ask what causes this, what is the nurse’s best response?

ORDER  HERE A PLAGIARISM-FREE PAPER HERE

nurs 6521 week 10 assignment

Application: Cancer and Women’s and Men’s Health

The American Cancer Society estimates that by the end of 2012, more than 226,000 women will be diagnosed with breast cancer and more than 241,000 men will be diagnosed with prostate cancer (American Cancer Society, 2012a; American Cancer Society 2012b). With such prevalence of women’s and men’s cancers, patient education and preventive services are essential. In clinical settings, advanced practice nurses must assist physicians in educating patients on risk factors, preventive services, and for patients diagnosed with cancer, on potential drug treatments. The clinical implications of women’s and men’s cancer greatly depend on early detection, which is primarily achieved through preventive services. In this Assignment, you consider the short-term and long-term implications of cancer and drug treatments associated with women’s and men’s health, as well as appropriate preventive services.

To prepare:

· Select a type of cancer associated with women’s or men’s health such as breast, cervical, or ovarian cancer in women and prostate cancer in men.

· Locate and review articles examining the type of cancer you selected.

· Review the U.S. Preventive Services Task Force article in the Learning Resources. Think about available preventive services that providers might recommend for patients at risk of this type of cancer.

· Select two of the following factors: genetics, gender, ethnicity, age, or behavior. Reflect on how these factors might impact decisions related to preventive services.

· Consider drug treatment options for patients diagnosed with the type of cancer you selected including short-term and long-term implications of the treatments.

To complete:

Write a 2- to 3- page paper that addresses the following:

· Describe available preventive services that providers might recommend for patients at risk of the type of cancer you selected.

· Select two of the following factors: genetics, gender, ethnicity, age, or behavior Explain how the factors you selected might impact decisions related to preventive services.

· Thoroughly Describe drug treatment for patients diagnosed with the type of cancer you selected. Explain the short-term and long-term implications of these treatments.

Reading

U.S. Preventive Services Task Force. (2014). The Guide to Clinical Preventive Services: Section 2. Recommendations for Adults. Retrieved from http://www.ahrq.gov/professionals/clinicians-providers/guidelines-recommendations/guide/section2.html

nurs 6521 week 11 assignment

NURS 6521 Final Exam – Question and Answers

1. Which of the following would the nurse include in a teaching plan about the signs and symptoms of thrombophlebitis and thromboembolism that should be reported by a patient taking estrogen?

2. A 38-year-old pregnant patient admits to the nurse that she is an alcoholic and has been consuming alcohol during her pregnancy. The nurse knows that using alcohol during pregnancy may result in a child who presents with

3. A nurse practitioner orders 150 mg of oral fluconazole for a patient with vulvovaginal candidiasis. The patient should expect to take medication

4. A nurse works at a weight management clinic. To which of the following overweight patients could the nurse safely administer dextroamphetamine?

5. A 71-year-old man has just been prescribed finasteride (Proscar). Which of the following complaints by this patient most likely indicated a need for this drug?

6. After 6 months of unsuccessfully trying to conceive, a 31-year-old woman and her husband have sought a referral to a fertility specialist in order to explore their options. A nurse at the clinic should recognize that the woman may benefit from

7. An oncology nurse is reviewing the pathophysiology of cancer and is discussing with a colleague the factors that contribute to the success or failure of a patient’s chemotherapy. Which of the following cancerous cells is most susceptible to the effects of chemotherapeutic drugs?

8. A middle-aged patient has received a diagnosis of GI stromal tumor following an extensive diagnostic workup. Imatinib has been recommended as a component of the patient’s drug regimen. What patient education should the nurse provide to this patient?

9. A 12-year-old boy is being discharged from the hospital after major surgery. The boy will be taking two medications at home for an extended period. The nurse who is discharging the patient should provide medication teaching specifically to

10. A patient has been admitted to the critical care unit with a diagnosis of peritonitis that has necessitated treatment with gentamicin. As a result, the care team should be cautious when concurrently administering other medications that may cause

11. A 39-year-old woman is receiving doxorubicin for the treatment of cancer. After each treatment, the patient has acute nausea and vomiting accompanied by a slightly increased heart rate. The nurse will advise her to

12. A nurse is caring for a patient with cancer who has been prescribed dronabinol (Marinol) to help reduce nausea and vomiting from chemotherapy. The nurse will inform the patient that he or she is taking an oral form of

13. A 46-year-old man is receiving a quinupristin/dalfopristin IV infusion for a life-threatening infection. Which of the following would be most important for the nurse to monitor?

14. A woman is receiving prolonged drug therapy during her complicated pregnancy, and it may pose a risk to both the mother and the fetus. The primary care physician has made dosage adjustments to minimize adverse effects and prevent toxicity. The nurse should make sure

15. Alprostadil (Caverject), a drug used to treat erectile dysfunction, has been prescribed to a 42-year-old patient. When providing education to the patient and his wife, the nurse should inform the wife about which of the following adverse effects?

16. A 36-year-old patient comes to the clinic and tells the nurse that she suspects that she is pregnant. During the initial assessment, the nurse learns that the patient is currently taking medications for diabetes, hypertension, and a seizure disorder. The nurse would be most concerned about which of the following medications?

17. A male patient with a medical background tells the nurse that he is not satisfied with the oral synthetic testosterone that has been prescribed for him and he would like to try a natural form of oral testosterone. Which of the following would be an appropriate response by the nurse?

18. A male patient is trying to decide if he should use finasteride (Proscar) to treat benign prostatic hypertrophy (BPH). When providing information about the drug, the nurse will include which of the following as a risk associated with finasteride therapy?

19. A 2-year-old child is diagnosed with a minor ailment and is to be administered medications at home for 2 weeks. The child lives with his mother, grandmother, and four other children between the ages of 14 months and 7 years. The home health nurse is asked to assess the home environment to determine if it is appropriate for the child to take his medication at home. Which of the following will have the greatest impact on the nurse’s assessment?

20. A 22-year-old woman has given birth to an infant who exhibits the signs and symptoms of maternal cocaine use during pregnancy. These signs and symptoms are a result of what pathophysiological effect of opioid use during pregnancy?

21. A 62-year-old patient taking tamoxifen exhibits increased bone and tumor pain along with a local disease flare. The nurse interprets this as an indication of which of the following?

22. A female patient is prescribed oprelvekin therapy to treat thrombocytopenia. Which of the following should the nurse continuously monitor to determine the efficacy and duration of the oprelvekin therapy?

23. The clinical nurse educator who oversees the emergency department in a children’s hospital has launched an awareness program aimed at reducing drug errors. What measure addresses the most common cause of incorrect doses in the care of infants and children?

24. A patient has completed 4 weeks of treatment with epoetin alfa. Which of the following assessment findings would most strongly indicate that treatment has been effective?

25. A patient has been scheduled to begin treatment with rituximab for non-Hodgkin’s lymphoma. The nurse who will administer this drug should understand that it targets

26. A 15-year-old boy has been diagnosed with bone cancer after several months of fatigue and pain. What question should the nurse include in an assessment when trying to minimize the potential for adverse drug reactions?

27. A 30-year-old man with a BMI of 59 has recently been diagnosed with type 2 diabetes mellitus. In light of the man’s lack of success with weight loss programs in the past, his care provider has prescribed sibutramine (Meridia). What instructions should the nurse consequently provide to this patient?

28. A 49-year-old farmer who normally enjoys good health has become seriously ill in recent days and the results of an extensive diagnostic work up have resulted in a diagnosis of histoplasmosis. The patient has been admitted to the hospital and has begun treatment with amphotericin B. The nurse who is providing care for the patient should prioritize which of the following diagnostic results during his course of treatment?

29. A patient has just received her first dose of imatinib and the nurse on the oncology unit is amending the patient’s care plan accordingly. What nursing diagnosis is most appropriate in light of this addition to the patient’s drug regimen?

30. A 20-year-old female patient is receiving topical clindamycin for acne vulgaris. She develops a rash and urticaria along with severe itching where the medication is applied. The nurse will formulate which of the following nursing diagnoses for the patient?

31. Which of the following nursing actions is most important in achieving successful antimicrobial therapy with vancomycin?

32. A woman who is in the second trimester of her first pregnancy has been experiencing frequent headaches and has sought advice from her nurse practitioner about safe treatment options. What analgesic can the nurse most safely recommend?

33. A patient receiving high-dose cisplatin therapy exhibits symptoms of hypomagnesemia. Which of the following should the nurse suggest to help offset the magnesium losses from the cisplatin therapy?

34. A child is admitted to the burn unit with second and third degree burns on both arms and part of his or her face. When administering topical medications to the burned areas, the nurse should

35. An immunocompromised 7-year-old child was recently discharged home with a peripherally-inserted central line (PIC line) for home antibiotic therapy. He has now been brought to the emergency department by his mother and father with signs and symptoms of line sepsis. Upon questioning, the mother states that she has been removing the PIC dressing daily and washing the site with warm water and a cloth. What nursing diagnosis is most appropriate in this situation?

36. A 21-year-old female has a history of irregular menses. She recently became sexually active, and would like to begin taking oral contraceptives (OCs). The nurse practitioner recognizes that most likely this patient would benefit from taking which category of OCs.

37. A 3-year-old boy has developed otitis media and requires antibiotics. In order to increase the chance that the boy will take his prescribed medication, the nurse should

38. A nurse is providing patient education to a 13-year-old girl who was just diagnosed with type 1 diabetes mellitus. Which of the following statements by the patient will alert the nurse that special instructions regarding insulin are necessary?

39. A 28-year-old woman has completed rituximab therapy for an autoimmune disease. She tells the nurse that she and her husband would like to start a family. The nurse will advise her to

40. A 73-year-old woman has osteoporosis and is prescribed alendronate. She takes calcium and vitamin D supplements, drinks lots of water, and has just quit smoking. The nurse should advise the patient to also

41. A nurse is performing patient education for a woman who has just been prescribed a bisphosphonate. Which of the following diagnostic and history findings would have prompted the woman’s care provider to prescribe a bisphosphonate?

42. A 20-year-old woman will soon begin taking oral contraceptives for the first time. What advice should the nurse provide to this patient?

43. A nurse is providing patient education to a 50-year-old woman who is taking methotrexate (MTX) for breast cancer. The nurse will instruct the patient to avoid which of the following drugs?

44. A nurse is instructing a 19-year-old female patient on the use of fluconazole for candida vaginitis. A teaching priority will be to

45. A nurse is assessing a patient who has chronic lymphoblastic myelogenous leukemia. The treatment plan includes hydroxyurea (Hydrea). The nurse will assess the patient for which of the following?

46. Mr. Singh is a 66-year-old man who is receiving chemotherapy for the treatment of lung cancer that has metastasized to his liver. In an effort to prevent infection, Mr. Singh has been prescribed filgrastim (Neupogen). Which of the nurse’s following assessment questions most directly addresses a common adverse effect of filgrastim?

47. A preterm neonate received caffeine for the treatment of apnea. The nurse should monitor the neonate for which of the following?

48. A patient has been prescribed daptomycin for a complicated skin infection. Which of the following will the nurse advise the patient to report immediately?

49. A 7-year-old child has been taking tetracycline for a bacterial infection. The nurse will be sure to inform the parents that this drug could cause

50. Sulconazole has been prescribed for a patient with tinea pedis. The nurse will instruct the patient to use the topical agent

51. A nurse has questioned why a patient’s physician has prescribed a narrow-spectrum antibiotic rather than a broad-spectrum drug in the treatment of a patient’s infection. Which of the following facts provides the best rationale for the use of narrow-spectrum antibiotics whenever possible?

52. A female patient has been prescribed estrogen therapy. Which of the following will the nurse advise the patient is a common adverse effect of estrogen therapy?

53. A nurse is discussing oprelvekin therapy with a male patient. Which of the following will the nurse tell the patient is the most common adverse effect of the drug?

54. A man has a demonstrated history of androgen deficiency and the consequences of this health problem include an inability to maintain an erection. Which of the following medications would best address this patient’s erectile dysfunction (ED)?

55. A female patient has follicular non-Hodgkin’s lymphoma and is receiving thalidomide (Thalomid). It will be most important for the nurse to monitor this patient for which of the following?

56. A nurse has administered filgrastim to a diverse group of patients in recent months. Which of the following patients should the nurse observe for extremely elevated white blood cell counts following administration of the drug?

57. A nurse is aware that the concept of selective toxicity is foundational to antimicrobial therapy. Which of the following statements most accurately describes selective toxicity?

58. A 59-year-old man with a recent history of erectile dysfunction has been assessed and prescribed sildenafil (Viagra). When providing patient education to this man, the nurse should tell him which of the following?

59. A nurse has completed a medication reconciliation of a patient who has been admitted following a motor vehicle accident. Among the many drugs that the patient has received in the previous year is rituximab. The nurse would be justified in suspecting the patient may have received treatment for which of the following diseases?

60. A patient asks the nurse practitioner about food sources such as soybeans and soy products. The nurse practitioner understands that these foods are considered

61. A nurse is providing patient education to a 23-year-old woman who is starting the norelgestromin/ethinyl estradiol transdermal system (Ortho Evra). Because this is the patient’s first time to use the birth control patch, the nurse will instruct her to apply the patch

62. A nurse is explaining the use of acyclovir therapy to a 72-year-old man. Nephrotoxicity is discussed as a major adverse effect in older patients. To minimize the risk of the patient developing this adverse effect, the nurse will advise him to

63. Which of the following patients will be at the greatest risk for anemia and would be the most likely candidate for epoetin alfa therapy?

64. A 35-year-old woman is on a weight-loss program and is to begin taking sibutramine (Meridia). After baseline physical data are obtained, the nurse will assess the patient’s childbearing potential. The nurse will inform the patient that during sibutramine therapy she should

65. The nurse recognizes that the potential for teratogenic drug effects is not static throughout the prenatal and postnatal periods. The potential for teratogenic effects is highest during

66. On the advice of her sister, a 52-year-old woman has visited her nurse practitioner to discuss the potential benefits of hormone replacement therapy in controlling the symptoms of menopause. Which of the following responses by the nurse is most appropriate?

67. A 16-year-old boy is prescribed cromolyn sodium nasal spray to treat a nasal allergy. To maximize the therapeutic effects of the drug, which of the following will the nurse include in instructions to the patient?

68. A nurse who provides care on a pediatric unit of a hospital is aware that the potential for harm as a result of drug errors is higher among infants and children than adults. This fact is primarily due to

69. A nurse who provides care on a pediatric medicine unit has conducted a medication reconciliation of a recently-admitted patient. In light of the fact that the child takes methylphenidate (Ritalin), the nurse is justified in considering a history of what health problem?

70. A nurse is caring for a 64-year-old female patient who is receiving IV heparin and reports bleeding from her gums. The nurse checks the patient’s laboratory test results and finds that she has a very high aPTT. The nurse anticipates that which of the following drugs may be ordered?

71. A 10-year-old boy is taking dextroamphetamine (Dexedrine) daily for ADHD. At each clinic visit, the nurse’s priority assessment would be

72. A 9-year-old boy was bought to his primary care provider by his mother with signs and symptoms of hookworm infection and will be sent home with a prescription for mebendazole. When provided patient and family education, the nurse should teach the mother with which of the following measures to avoid reinfection following treatment?

73. A 43-year-old man has been diagnosed with active TB. He is prescribed a multiple drug therapy, including INH and rifampin. A priority assessment by the nurse will be to monitor which combination of laboratory test results?

74. During ongoing assessment of a patient receiving 5-FU therapy, the nurse finds the patient’s platelet count to be 92,000 cells/mm3. The nurse should do which of the following?

75. A nurse is discussing with a 58-year-old male patient the causes of erectile dysfunction in men over 50 years of age. Which of the following will the nurse inform the patient is the primary physical cause of erectile dysfunction of men in this age group?

76. A patient has been admitted to the critical care unit of the hospital with bacterial septicemia that has failed to respond to initial antibiotic treatment. The patient’s most recent blood cultures reveal the presence of methicillin-resistant Staphylococcus aureus (MRSA) in the patient’s blood. The nurse will anticipate that this patient will likely require intravenous administration of what antibiotic?

77. A nurse is instructing a colleague on how an antimicrobial produces a therapeutic effect. Which of the following should be included in the nurse’s teaching?

78. A 20-year-old woman has been prescribed estrogen. As with all women taking estrogen, the nurse will carefully monitor the patient for which of the following?

79. A nurse has been assigned to a 55-year-old woman who has a malignant brain tumor. The patient is receiving her first dose of carmustine. It will be critical for the nurse to observe for which of the following?

80. A man is prescribed ciprofloxacin to treat a sexually transmitted infection. The nurse will instruct the patient to

81. A 66-year-old man has made an appointment with his primary care provider to discuss his recent erectile dysfunction (ED) and has requested a prescription for tadalafil (Cialis) based on television commercials he has seen. What characteristic of this patient would most likely contraindicate the use of tadalafil for his ED?

82. A nurse is caring for a patient who is at 28 weeks’ gestation and is receiving terbutaline (Brethine) to control preterm labor. Which of the following assessment parameters should the nurse prioritize?

83. A patient with non-Hodgkin’s lymphoma (NHL) will be starting a course of doxorubicin shortly. When planning this patient’s care, what nursing diagnosis should the nurse prioritize?

84. A patient is taking rifampin (Rifadin) for active TB. When discussing this drug with the patient, the nurse should stress that

85. A nurse is administering rituximab to a patient via the IV route. The nurse will set the IV pump at 50 mg/hour for the first half hour of the initial infusion. If there are not apparent reactions after 30 minutes of the infusion, the nurse will increase the dosage every 30 minutes by 50 mg/hour until the maximum infusion rate reaches which of the following?

86. A 60-year-old patient experienced a sudden onset of chest pain and shortness of breath and was subsequently diagnosed with a pulmonary embolism in the emergency department. The patient has been started on an intravenous heparin infusion. How does this drug achieve therapeutic effect?

87. A patient will soon begin targeted therapy as a component of her treatment plan for chronic leukemia. The nurse is conducting health education about this new aspect of the patient’s drug regimen and the patient has asked about the potential side effects of treatment. How should the nurse best respond?

88. A patient is taking etoposide for a testicular tumor refractory to treatment. The nursing assessment reveals that he is also taking warfarin. The nurse must carefully monitor for which of the following?

89. A patient is receiving long-term clindamycin therapy for a life-threatening infection. The nurse will begin by monitoring this drug therapy by obtaining

90. A patient is prescribed ganciclovir to treat a CMV infection. An oral dosage is prescribed. To help increase bioavailability of the drug, the nurse will encourage the patient to take the medication

91. A 29-year-old woman who is morbidly obese has recently begun a comprehensive, medically-supervised program of weight reduction. Prior to adding dextroamphetamine (Dexedrine) to her regimen, the patient should be questioned about her intake of

92. A patient is pregnant and is at 7 weeks’ gestation. She has type 1 diabetes and has been taking insulin since she was 13 years old. She asks the nurse if the insulin will be harmful to her baby. The best response to the patient by the nurse would be

93. A nurse is explaining to the parents of a 6-year-old child suffering from angina why nitroglycerin patches for chest pain would not be appropriate. Which of the following will the nurse include in an explanation?

94. Mr. Lepp is a 63-year-old man who was diagnosed with colon cancer several weeks ago and who is scheduled to begin chemotherapy. He reports to the nurse that he read about the need for erythropoietin in an online forum for cancer patients and wants to explore the use of epoetin alfa with his oncologist. Which of the following facts should underlie the nurse’s response to Mr. Lepp?

95. A patient with a diagnosis of chronic myeloid leukemia has met with her oncologist, who has recommended treatment with the kinase inhibitor imatinib. What route of administration should the nurse explain to the patient?

96. A nurse is going to administer medication to an infant using a medicine dropper. The best method is to open the child’s mouth by gently squeezing the cheeks and placing the drops

97. A nurse practitioner orders a single dose of 2 g Metronidazole orally. How many milligrams will the patient receive in one dose?

98. A patient is being treated for Mycoplasma pneumoniae pneumonia. She is allergic to penicillin and is being given azithromycin (Zithromax) in capsule form. The nurse will inform the patient that she will need to take the capsule

99. A nurse is explaining to a pregnant 21-year-old college student why she cannot continue to take ibuprofen (Advil) for her headaches. The nurse draws a picture depicting drug molecules crossing the placental membrane and entering into the fetal circulation. The nurse tells the patient that the main reason this happens is because

100. A patient has been prescribed oral tetracycline. The nurse will instruct the patient to take the drug NURS 6521 Assignments Essay